pocket prep 2/2

Pataasin ang iyong marka sa homework at exams ngayon gamit ang Quizwiz!

Which of the following is not commonly asked of a patient during an orientation evaluation?

"Would you please tell me who is president of the United States?" Orientation is a patient's ability to state their name and the time and to describe their surroundings. Common things asked of a patient are their name; the current year, month, or approximate date; and the current location. Avoid asking, "Do you know..." questions, as they may elicit a "yes" answer and not establish orientation. Although it is sometimes used in more detailed mental status exams, asking the identity of the current president is not a correct question for establishing a patient's level of orientation.

What is an EMT's primary concern?

-Personal safety -Personal safety is of the utmost importance in any scenario. Whenever possible, do not enter a potentially dangerous situation. Law enforcement should be contacted if violence is possible. Law enforcement should secure a scene prior to EMS entrance.

What is the correct supplemental oxygen flow rate for a non-rebreather mask?

10-15 liters per minute The correct supplemental oxygen flow rate for a non-rebreather mask is 10-15 liters per minute. At this rate and with a reservoir bag, up to 95% oxygen may be delivered. Flow rates for various devices are as follows: nasal cannula: 2-6 L/min (delivers FiO2 of 24-44%) simple mask: 6-10 L/min (delivers FiO2 of 40-70% non-rebreather: 10-15 L/min (delivers FiO2 of 66-95%) bag-valve mask: 10-15 L/min (delivers FiO2 nearly 100%)

What is the average length of the first stage of labor in a primigravida patient?

12-18 hours The average length of the first stage of labor (the dilation of the cervix) in a primigravida patient is 12-18 hours. Typically, a patient in the first stage of labor, regardless of the number of prior pregnancies, will not deliver prior to transport.

What is the maximum percentage of oxygen that can be delivered via a nasal cannula at a maximum flow rate of six liters per minute

44% At its maximum flow rate (six liters per minute), a nasal cannula can deliver 44% oxygen. A bag-valve mask with a reservoir bag with supplemental oxygen flowing at 15 liters per minute will provide nearly 100% oxygen to a patient. Assisted ventilation, like a bag-valve mask, is indicated for patients who are not breathing adequately or are in respiratory distress or respiratory failure. A mouth-to-mask, another form of assisted ventilation with supplemental oxygen flowing at 15 liters per minute, will provide only 55% oxygen. A non-rebreather mask with a reservoir bag with supplemental oxygen flowing at fifteen liters per minute will provide up to 95% oxygen to a patient. A non-rebreather mask should be used for patients with adequate breathing and suspected or confirmed hypoxia.

Which of the following are the signs and symptoms of a hypertensive emergency?

A hypertensive emergency is defined as systolic blood pressure greater than 180 mmHg with impending or progressive organ damage. Signs and symptoms of a hypertensive crisis include: ringing in the ears severe headache with dizziness bounding pulse nosebleed altered mental status warm skin that may be flushed and moist Cool, clammy skin is a sign of low blood pressure.

Which of the following are devices that are appropriate to provide spinal motion restriction for a patient with a suspected spinal injury?

An EMT can use several devices to provide spinal motion restriction, depending on local protocols. Options include spine boards, the KED device, and a full-body vacuum mattress. A traction splint is used for isolated femur fractures, not spinal motion restriction.

The administration of nitroglycerin for cardiac-related chest pain can have strong benefits for a patient experiencing acute coronary syndrome (ACS). Which of the following statements is correct regarding the administration of nitroglycerin?

An EMT may administer nitroglycerin after obtaining an order from medical control and ensuring the patient's medication rights. Always follow local protocols. Guidelines for nitroglycerin administration include: Do not give nitroglycerin if the systolic BP is below 90 mmHg. Do not give nitroglycerin if the maximum dose has been given, usually 3 doses. Do not give nitroglycerin if the patient has taken erectile dysfunction drugs within the previous 48 hours. Nitroglycerin loses potency over time, and expired medication will most likely not be as effective. A headache is a common side effect of nitroglycerin. Patients may report a fizzing sensation when a nitroglycerin tablet is placed under the tongue.

A 30-year-old female was found in a city park and is having trouble breathing. She complains of steadily increasing shortness of breath and urticaria and feels as if her throat or tongue is swelling. Which of the following is the most likely cause of her signs and symptoms?

Anaphylactic shock Anaphylaxis, or anaphylactic shock, is a violent reaction to an allergen. Common causes of anaphylaxis include injections (e.g., tetanus antitoxin, penicillin), stings from insects, ingestion of foods, and inhalation of particles (e.g., dust, pollen, mold). It may develop within seconds or minutes of contact with the allergen. A repeat phase 1-8 hours after exposure is possible. Flushed, itchy skin, urticaria (hives), edema, pallor, cyanosis, hypotension, weak or almost impalpable pulse, sneezing, stridor/airway obstruction, chest tightness, dyspnea, coughing, wheezing, respiratory arrest, abdominal cramping, nausea/vomiting, altered mental status, dizziness, syncope, and coma are possible. Asthma is an acute spasm of bronchioles associated with excessive mucus production and swelling of the respiratory mucosal lining. Wheezing, dyspnea, cyanosis, and respiratory arrest may occur. Urticaria is not commonly associated with asthma. Syncope is fainting (a near or brief loss of consciousness) and can be a sign of many medical emergencies. Heat exhaustion is the most common heat emergency. It is caused by heat exposure, stress, and fatigue. Hypovolemia is common due to the loss of water and electrolytes via sweating. Dizziness, weakness, syncope, muscle cramping, a dry tongue/thirst, and tachycardia are common.

Which of the following is the most appropriate way to control external bleeding caused by damage to a large artery or vein?

Apply direct pressure to the wound Applying direct pressure to the site of a wound will control most external bleeding and should be attempted first. If direct pressure fails to immediately stop the hemorrhage, apply a tourniquet above the level of bleeding. If this is not possible, consider a junctional tourniquet or wound packing with a hemostatic agent (if protocols allow). Pressure dressings can be applied once bleeding is controlled. They were once recommended for control of hemorrhage, but evidence indicates a tourniquet is more effective. Elevation of an extremity was previously recommended, but evidence indicates it does not control bleeding.

Which of the following statements are correct regarding AED use on an adult cardiac arrest patient?

Automated external defibrillator (AED) use in cardiac arrest improves patient outcomes. The AHA recommendations on AED use include the following: Ensure adequate CPR is being performed while the AED is being powered up and placed. Limit any interruption of CPR to ≤10 seconds. Power on the AED and apply pads. Stop CPR briefly when the machine advises or push the analyze button (if present). If a shock is advised, clear the patient and deliver the shock; after the shock is delivered, immediately resume CPR. If no shock is advised, resume CPR and check the pulse when directed by the AED or after 5 cycles of CPR. CPR can be delivered while the AED is charging for a shock; ensure the patient is clear before delivering a shock. Gather information about the arrest event if possible. If cardiac arrest occurs during transport, stop the ambulance while analyzing and shocking the patient with an AED.

Which pulse point is best for assessing an unresponsive infant?

Brachial pulse A brachial pulse is assessed in an infant to determine pulse rate. CPR should be initiated on an infant without a palpable pulse or a pulse below 60 beats per minute. A carotid or a femoral pulse is not typically palpable in a child under one year of age. The radial pulse is not used to assess the need for CPR in any age group.

Which of the following are signs that a spinal injury patient is in neurogenic shock? Select the three answer options which are correct.

Classic signs of neurogenic shock include hypotension; bradycardia; warm, flushed skin below the level of the spinal cord injury (due to loss of sympathetic tone); and cool, clammy skin above the level of injury. Patients may also develop a loss of bladder control, priapism, and paralysis of the small bowel

In which of the following scenarios would eye protection be required?

Controlling arterial bleeding that is spurting bright red blood Eye protection and face shields should be used any time bodily fluids could splatter. This potential exists primarily in cases of uncontrolled arterial bleeding, active vomiting, and suctioning. Caring for a sweaty patient, caring for a patient covered in a substance, or handling contaminated instruments may not necessitate eye protection or a face shield. The use of personal protective equipment should meet the needs of each incident. Standard precautions should be used consistently.

Which of the following results from infections and will most likely cause respiratory distress in pediatric patients?

Croup is a viral infection of the upper airway. Epiglottitis is swelling of the epiglottis that results from an upper respiratory infection. Bacterial tracheitis is an acute bacterial infection of the subglottic area of the upper airway. All of these conditions are likely to cause respiratory distress in pediatric patients. Asthma is a reactive airway disease and is not caused by an infection. It also leads to respiratory distress in pediatric patients.

Which of the following signs/symptoms would not be associated with acute coronary syndrome?

Diarrhea Acute coronary syndrome is a group of symptoms caused by myocardial ischemia. Angina pectoris (stable and unstable), as well as myocardial infarction, falls under acute coronary syndrome. Angina pectoris and myocardial infarction are indistinguishable in a prehospital setting. Chest pain, described as crushing, squeezing, or pressure, is a common symptom. This pain may radiate to the jaw, arms, epigastrium, or back. Dyspnea, weakness, nausea/vomiting, irregular heartbeat, diaphoresis, and syncope often occur.

Which of the following defines dysphagia?

Difficulty swallowing Dysphagia can mean difficulty swallowing or eating, depending on the context. Dyspnea is difficulty breathing. Common causes of dyspnea include pulmonary edema, hay fever, pleural effusion, airway obstruction, hyperventilation syndrome, environmental/industrial exposure, carbon monoxide poisoning, and drug overdose. Dysuria is difficult or painful urination. Difficulty speaking is referred to as dysarthria.

Which of the following situations constitutes a breach of confidentiality?

Discussing the treatment provided on a call in a public area Confidential information includes patient information (e.g., name, date of birth, demographics) and history, assessment findings, and treatment provided. If this information is disclosed without appropriate authorization, it may constitute a breach of confidentiality. Commonly, a subpoena or signed written release from the patient is required for the release of patient information. Third-party billing personnel also have access to patient information. Discussing patient information with receiving hospital personnel is appropriate and part of the continuity of care. If information is known about a patient or scene prior to arrival, this information may help all involved parties to prepare and ensure appropriate scene safety. Preparation, both mentally and physically, will help alleviate potential dangers upon entering a scene.

A 7-year-old patient has a history of severe allergic reactions. He is currently experiencing facial swelling, urticaria, and inspiratory/expiratory wheezing that is equal bilaterally. If available, which intervention would be the most appropriate to initiate?

Epinephrine auto-injector This patient is showing signs of a severe allergic reaction with multiple system involvement—anaphylaxis—which is consistent with his past medical history. Anaphylaxis can cause bronchospasms, wheezing, chest tightness, coughing, dyspnea, hypotension, anxiety, and gastrointestinal complaints. Epinephrine is a hormone given intramuscularly to treat anaphylaxis. Auto-injectors are dosed at 0.3 mg for patients three years and older and 0.15 mg for patients under three years of age.

An EMT is under noticeable stress; their mood has changed, and they fall asleep at work. Which of the following is a potential activity that could be exacerbating their response to stress?

Excessive alcohol use Acceptable alcohol use is currently defined as 1-2 drinks per day (1 for women and 2 for men). Avoidance of alcohol may help manage stress. Some other strategies for managing stress include the following: minimize/eliminate stressors as much as possible change partners to avoid a negative/hostile personality change work hours or environment cut back on overtime change attitude about a stressor talk about feelings with trusted people seek professional counseling, if needed do not obsess over frustrating situations that are not able to be changed adopt a relaxed, philosophical outlook expand friends and social support systems and develop interests beyond emergency services perform relaxation techniques (e.g., yoga, deep breathing, muscle relaxation, meditation) perform regular physical exercise limit intake of caffeine and tobacco Proper nutrition, hydration, exercise, relaxation, and sleep are important to prepare for potentially stressful situations. Additionally, balancing work, family, and health can prevent future stressors and alleviate current stress.

Which of the following terms is not used to characterize a pulse?

Full Pulse quality and rhythm characterize a person's pulse. A pulse that is strong with a regular rhythm is considered normal. Bounding refers to a pulse that is stronger than normal. Thready or weak refers to a pulse that is not as strong as normal. An irregular rhythm, or a rhythm that is too fast or too slow, is considered abnormal. Full is a term that describes the vasodilation of a blood vessel. It is not used to describe a normal pulse.

Which of the following is not part of the SAMPLE mnemonic?

Immunization status Explanations for each letter are as follows: Signs and symptoms: symptoms that occurred before or during the onset or are actively occurring, OPQRST for pain Allergies: allergies to medication, food, or other substances; reactions to allergens Medications: any medications, dosing, frequency, last time taken Pertinent medical history: previous medical, surgical, or traumatic occurrences that pertain to the current injury/illness Last oral intake: what and when the patient ate or drank, drug or alcohol use Events leading to injury/illness: time of onset, activities performed at onset

Which of the following principles allows an emergency medical technician to treat and transport an adult patient who is intoxicated?

Implied consent Implied consent is assumed if a patient is unconscious or otherwise incapable of making a rational, informed decision. Examples of this are patients who are intoxicated by drugs or alcohol, mentally impaired, or suffering from a condition that precludes making a rational, informed decision (e.g., head injury). Implied consent only applies when a serious medical condition exists (i.e., a threat to life or limb). Every effort should be made to obtain consent from an available relative prior to treatment, but lifesaving care should not be delayed while waiting for such consent. Expressed consent is the verbal authorization for, or otherwise acknowledgment of, treatment and/or transport. For consent to be legitimate, there must also be informed consent. Informed consent is given after the patient understands the risks, benefits, and alternatives to treatment/transport, as well as the consequences of the refusal of treatment. An advance directive is a written document that specifies medical treatment for a competent patient if they become unable to make decisions.

If injured, which of the following will likely result in the greatest blood loss?

Liver Solid organs (e.g., the liver, spleen, diaphragm, kidneys, and pancreas) can bleed significantly and cause rapid blood loss. Abdominal pain is common with internal bleeding from solid organ injury. However, each organ will exhibit different signs/symptoms if damaged. The risk of internal bleeding is much greater with an injury to solid organs such as the liver. Hollow organs, such as the gallbladder, stomach, urinary bladder, and small and large intestine (including the appendix and rectum), are more likely to lead to infection if damaged.

Which of the following may be indicated for an adult patient with pulmonary edema? Select the two answer options which are correct.

Nitroglycerin is highly beneficial when used for patients with pulmonary edema, as it leads to a reduction in arteriolar pressure that decreases hydrostatic pressure in the capillary bed. This reduces fluid being forced from the capillaries into the lungs. Follow local protocols and contact medical control for orders for nitroglycerin administration. CPAP treats pulmonary edema by increasing pressure in the alveoli, forcing fluid back into the interstitial space and out of the lungs. Always request ALS intercept when treating a patient with acute pulmonary edema. Albuterol is a bronchodilator used frequently to reverse bronchospasm, but it has little effect on pulmonary edema. Aspirin is used in acute coronary syndromes to reduce thrombosis and slow the progression of myocardial infarction. It is not useful for managing pulmonary edema.

Which of the following is not a cause of vaginal bleeding in pregnancy?

Preeclampsia Preeclampsia is a possible complication during pregnancy, typically affecting primigravida patients. It usually develops after the 20th week of gestation and is characterized by severe hypertension, proteinuria, headaches, visual abnormalities, edema, and anxiety. Ectopic pregnancy (when an embryo develops outside of the uterus), spontaneous abortion (miscarriage), and abruptio placentae (separation of the placenta prematurely from the uterine wall) are all potential causes of bleeding during pregnancy.

A 12-year-old male was stung by a bee. He still has the stinger attached, so his mother calls for assistance in removing it. What is the proper procedure for removing a bee stinger that is still partially embedded in a patient's skin?

Scrape it with a rigid object If a stinger is left in place, it may continue to inject venom for up to 20 minutes. To remove a stinger, gently scrape the skin with the edge of a sharp, stiff object (e.g., a credit card). Using tweezers or forceps or squeezing the skin may inject more venom into the wound.

Which findings characterize hypoglycemia?

Signs and symptoms of hypoglycemia include: rapid onset of symptoms altered mental status slow, shallow respirations when severe rapid, weak pulse (tachycardia) pale, cool, and moist skin

A 4-year-old female has suffered an injury and is very upset. Which of the following is an appropriate way to interact with this patient?

Tell the child what is going to be done immediately before doing it Preschool-aged children (three to six years) can use simple language effectively. Children at this age have a vivid imagination, so interventions should be explained simply and directly before they are done to ensure the child has no time to develop frightening fantasies. It is inappropriate to ignore any patient. It is also inappropriate to lie to any patient, particularly preschool-aged children, as their trust is hard to regain.

Which of the following statements are true regarding the glass on a vehicle? Select the 2 answer options which are correct.

Tempered glass is found in the side and rear windows of a vehicle and is most easily broken with a window punch. Safety glass is found in the front windshield and is not easily broken. A tool such as a glass-cutting saw is needed to cut the windshield, but in most cases, the windshield is removed in one piece because it is difficult to cut.

The NFPA 704 system is used to indicate hazards found in buildings. The components of NFPA 704 include which of the following? Select the three answer options which are correct.

The NFPA 704 system uses diamond-shaped symbols that contain four smaller diamonds, each representing a specific hazard. Red indicates a flammability hazard, blue indicates a health hazard, white provides special information, and yellow indicates a reactivity hazard. Each small diamond is rated on a scale from 0 (no risk) to 4 (high risk). The NFPA 704 system is used for hazard identification at fixed facilities, not transportation hazards. The DOT system is used for materials being transported.

Which of the following is not part of the minimum information collected by dispatch for an emergency request?

The ages of each patient Dispatch must be easy to access and in service for 24 hours a day. Operation responsibilities and jurisdiction will differ by location. Trained personnel who are familiar with the service area geography should provide dispatch services for their respective region. For each emergency request, a dispatcher should obtain and record the following: the nature of the call the name, present location, and call-back telephone number of the caller the location of the patient(s) Age of the patient Is the patient conscious and breathing the number of patients and estimations of the severity of their condition(s) other special problems or pertinent information (e.g., hazards, weather conditions)

The Stroke Chain of Survival described by the American Heart Association (AHA) and the American Stroke Association (ASA) includes which of the following? Select the 3 answer options which are correct.

The goal of stroke care is to minimize brain injury and maximize the patient's recovery. The Stroke Chain of Survival described by the AHA and ASA links the actions that patients, family members, and healthcare providers should take to maximize stroke recovery. These actions are: rapid recognition of and reaction to stroke warning signs and symptoms rapid use of 9-1-1 and EMS dispatch rapid EMS recognition of stroke, triage, transport, and pre-hospital notification to the receiving hospital rapid diagnosis and treatment in the hospital Rapid administration of fibrinolytic medications is not one of the links in the Stroke Chain of Survival.

A 16-year-old patient was struck in the mouth by a thrown baseball, knocking an incisor out of its socket. Which of the following is not true about the proper management of a permanent tooth avulsion?

The tooth should be rinsed immediately with copious amounts of water. Bleeding will occur when a tooth is violently displaced from its socket; therefore, direct pressure should be administered. Suctioning and/or removal of cracked or loose teeth may be required to maintain a patent airway. Handle any avulsed tooth by the crown, not the root. Place the tooth in a special tooth storage solution, if available, or in cold milk or sterile saline. Reimplantation is recommended from 20 minutes to one hour after the incident.

Which of the following is the leading cause of death in adolescents?

Unintentional injuries Unintentional injuries are the leading cause of death in adolescents, as well as in all patients ages one to 44 years. Suicide is the third-leading cause of death in adolescents. Congenital abnormalities are the leading cause of death in infants. Cancer is the leading cause of death in patients from 45 to 60 years old.

If an automated external defibrillator (AED) analyzes a shockable rhythm, which of the following cardiac dysrhythmias is most likely evident?

`: Ventricular fibrillation or ventricular tachycardia An automatic external defibrillator (AED) will advise a shock for ventricular fibrillation or ventricular tachycardia; rarely, a shock will be advised without one of these rhythms. Early defibrillation for these rhythms has been linked to an increased survival rate. Defibrillation is not a substitute for high-quality CPR but is a factor that promotes survival. Asystole ("flatline"), pulseless electrical activity, and atrial fibrillation and/or atrial flutter are not shockable rhythm

Which of the following are correct treatments for removing a foreign body airway obstruction in a responsive infant?

or a responsive infant who is choking, perform the following steps: Hold the infant facedown with the body resting on your forearm, supporting the jaw and face with your hand. Keep the head lower than the rest of the body. Deliver 5 back blows between the shoulder blades, using the heel of your hand. Place your free hand behind the infant's head and turn the infant face-up on your other arm, keeping the head lower than the rest of the body. Give 5 quick chest thrusts in the same location and manner as the chest compressions; using 2 fingers on the lower part of the sternum. Check the airway- if you can see a foreign body, then remove it. Do not perform blind finger sweeps. If you cannot see the object, repeat the cycle as many times as needed. If the infant becomes unresponsive, begin CPR.

A 56-year-old male patient with a cardiac history complains of substernal chest pain. Which question is asked to assess the "O" portion of the OPQRST mnemonic?

"What were you doing when the pain began?" The OPQRST mnemonic summarizes a way to gather background information on a patient's chief complaint. OPQRST stands for onset, provocation/palliation, quality, region/radiation, severity, and timing. Common questions for each area are as follows: Onset: "What were you doing when the symptoms began?" Provocation/Palliation: "Does anything make the symptoms better or worse?" Quality: "What does the symptom feel like?" Region/Radiation: "Where do you feel the symptom? Does it move anywhere?" Severity: "On a scale of zero to ten, how would you rate your symptom?" Timing: "When did the symptom start? How long has it been occurring? Has it been constant, or does it come and go?"

ALS providers are preparing to hyperventilate a patient with a suspected brainstem herniation. As waveform capnography becomes available, what should be the target end-tidal carbon dioxide level during hyperventilation?

30-35 mm Hg Hyperventilation is a controversial treatment that is sometimes used for patients who have a brainstem herniation. It should only be done if allowed by local protocols. Additionally, waveform capnography should be available to ensure that end-tidal carbon dioxide (ETCO2) levels are between 30 and 35 mm Hg while hyperventilating.

How long can the brain be deprived of oxygen before developing irreversible damage?

4-6 minutes A normal, healthy adult patient may begin to experience permanent brain damage after 4-6 minutes without oxygenation. Simple acts, such as opening an airway, may restore normal breathing in unconscious patients. If spontaneous breathing is not restored by opening the airway, the patient may require assisted ventilation.

What is the maximum oxygen concentration that can be delivered via a nasal cannula?

44% A nasal cannula will commonly deliver between 24% and 44% oxygen and has a flow rate range of one to six liters per minute. A nasal cannula should be reserved for patients who are unable to tolerate a non-rebreather mask. A non-rebreather mask with a reservoir of supplemental oxygen at 15 liters per minute can deliver oxygen up to 95%.

A patient has an impaled object in their right thigh. Which of the following is not appropriate for impaled object injuries in the extremities?

: Apply ice to help reduce swelling It is not appropriate to apply ice to an impaled object. This may actually impair blood clotting and make the wound bleed. Penetrating injuries generally have obvious wounds and external bleeding. Pain at the site of the wound, nausea, and vomiting are common. If a penetrating (impaled) object is still in place, apply a stabilizing bandage around it to control external bleeding and minimize the movement of the object. Inspect the patient's back in cases of anterior penetrating injuries and the front to assess for exit wounds. Removal of a penetrating object outside of a surgical setting may cause more injury to the area; therefore, in a prehospital setting, penetrating objects should remain in place unless a life-threatening condition (e.g., inability to manage the airway) exists.`

What is the anatomical location of the cricoid cartilage?

: Inferior to the thyroid cartilage, at the lowest portion of the larynx The cricoid cartilage is a firm ring that is located inferior to (below) the thyroid cartilage at the lowest portion of the larynx. The carina is the level at which the trachea divides into the two mainstem bronchi. The epiglottis appears at the base of the oropharynx, superior to (above) the cricoid cartilage.

Which of the following are leads placed on a patient's limbs to acquire a 12-lead ECG?

A 12-lead ECG is obtained by placing 10 electrodes on the patient: 4 electrodes are placed on the limbs, and 6 are placed on the chest. Electrodes LL (left leg), LA (left arm), RL (right leg), and RA (right arm) are placed on the limbs, and electrodes V1 through V6 are placed on the chest.

For which of the following patients is an emergency move appropriate?

A pulseless 54-year-old male slumped over a table An emergency move should be performed if there is a risk of serious harm or death (e.g., fire, explosives, hazardous materials), an inability to gain access to other patients who need lifesaving care, or life-threatening conditions that cannot be assessed and/or treated. A patient who is pulseless and not in a position in which CPR can be initiated would require an emergency move. A patient who is supine on the ground or on a flat surface can be assessed and treated adequately. A non-trauma, conscious patient may be able to move on their own; this should be attempted prior to an emergency move if the patient is unable to be assessed or treated.

A 56-year-old obese male reports left-sided chest pain that started when he was walking his dog. The patient describes the pain as "pressure" that has persisted for over an hour. He is short of breath and feels like he's sweating more than usual. The patient explains these episodes have happened before and usually improve with rest. He smoked 1.5 packs of cigarettes per day for 25 years but quit two months ago. He denies drug or alcohol use. A physical exam shows a pulse of 110 beats per minute, a respiratory rate of 18 breaths per minute, and a blood pressure of 188/96. What is the most likely diagnosis?

Acute coronary syndrome Chest pain is one of the most common emergency complaints with many possible causes. Some of the most emergent causes include acute coronary syndrome (ACS), aortic dissection, pericarditis/myocarditis, tension pneumothorax, pulmonary embolism, esophageal rupture, and perforated peptic ulcer. Features that increase the likelihood of ACS include substernal pain with a gradual onset that lasts for 30 minutes to 3 hours; the pain is frequently described as tightness or pressure ("like an elephant sitting on my chest"), crushing, or squeezing. Any patient presenting with symptoms of angina should be assessed for acute myocardial infarction (AMI). Most prehospital care for angina pectoris consists of administering nitroglycerin, oxygen, and aspirin. The availability of a prehospital ECG is becoming more common. Pulmonary embolism typically arises from a thrombus that originates in the deep veins of the lower extremities. An embolus dislodges from the thrombus, travels to the lung, and gets caught in an artery of the lung; large thrombi can lodge at the bifurcation of the main pulmonary artery (saddle embolus) or the lobar branches and cause severe hemodynamic collapse. Aortic dissection is defined as separation of the layers within the aortic wall. Tears in the intimal layer result in the propagation of dissection (proximally or distally) secondary to blood entering the intima-media space. Mortality is quite high. Patients often describe the sudden onset of severe chest pain that mimics AMI or a sudden onset of tearing or ripping pain between the shoulder blades, depending on the location of the dissection. Patients sometimes describe their pain as knifelike, sharp, or stabbing. The classic presentation of esophageal rupture is a middle-aged man with a history of alcohol abuse who experiences chest pain and subcutaneous emphysema after recent vomiting or retching. All of these conditions are life-threatening and require immediate intervention.

An automated external defibrillator (AED) is indicated in which of the following scenarios?

A 24-year-old female who is unresponsive and has no palpable pulse An AED is indicated for unresponsive and pulseless patients. Defibrillation is not a substitute for high-quality CPR but constitutes one of the five links in the chain of survival.

Which of the following patients is most likely to suffer from a heat emergency?

A 24-year-old male complaining of leg cramps after running several miles Athletes of any age and those who work in hot environments (e.g., firefighters, soldiers, construction workers) need to know the basics of physiologic thermoregulation, heat illnesses, and strategies for prevention and treatment. Risk factors that increase the likelihood of heat-related illnesses include a preceding viral infection, dehydration, fatigue, obesity, lack of sleep, and poor physical fitness. The use of illicit drugs such as cocaine and amphetamines (Ecstasy) also increases a person's risk. Athletes in good condition are still at risk of experiencing a heat emergency in warm and/or humid climates. Constricted pupils, decreased reflexes, and itchy skin are not associated with heat emergencies.

Which of the following patients requires rapid transport?

A 28-year-old male with an allergic reaction to a bee sting who has dyspnea and a rapid pulse High-priority patients require rapid transport. Examples of high-priority patients include those who are unresponsive or those who are responsive but unable to follow commands, yield a poor general impression, have difficulty breathing, have uncontrolled bleeding, have severe chest pain, have severe pain in any area of the body, have pale skin or signs of poor perfusion, or are pregnant with likely complicated childbirth. Crowning indicates that birth is imminent. Delivery should be considered at the scene.

Which of the following patients is most likely to experience cardiac compromise?

A 32-year-old man with an irregular pulse and dyspnea Chest pain/discomfort that is commonly described as pressure or heaviness, an irregular heart rate, syncope (fainting), nausea/vomiting, dyspnea, and sweating are common signs and symptoms of cardiac compromise. Other signs of cardiac compromise may be weakness, syncope, or altered mental status. Bradycardia (a pulse below 60 beats per minute) is common in athletes as a result of physiological changes related to physical conditioning. Difficulty breathing, wheezing, and urticaria are signs of anaphylaxis (a severe allergic reaction).

An elderly female is in respiratory distress. Gasping for breath, the patient attempts to explain the history of her respiratory problems, beginning in her youth. What is the appropriate response?

Advise the patient to focus on her current complaint In a prehospital setting, historical information that is not associated with the chief complaint does not need to be discussed. In the focused assessment, the information discussed and the assessment should be based on the chief complaint. In this scenario, excessive talking may lead to further dyspnea. Additionally, delaying the assessment/treatment for a complete history may cause further respiratory exacerbations.

For which of the following patients would the administration of oral glucose be indicated?

A 42-year-old male with a history of diabetes and sudden-onset altered mental status Oral glucose is indicated for patients with hypoglycemia. A patient who does not require oral glucose will not be harmed by it, so it should be given if hypoglycemia is suspected. Hypoglycemia will often cause a rapid onset of altered mental status, particularly in a patient with a history of diabetes. Other signs and symptoms include pale, cool, moist skin; hypotension; rapid, weak pulse; and possible shallow respirations in severe cases. Oral glucose should not be given to an unresponsive patient, or a patient without a gag reflex, due to the risk of aspiration. This 41-year-old likely has altered mental status due to the injury to his head. He does not require oral glucose at this time and should not be given anything by mouth. Hemiplegia is not a common sign or symptom of hypoglycemia. Stroke patients should not be given anything by mouth. This 28-year-old female is unresponsive, which is a contraindication to oral glucose. An unresponsive patient should not be given anything by mouth.

Nitroglycerin is an effective medication that helps reduce chest pain caused by ischemia. From which of the following patients should nitroglycerin be withheld?

A 55-year-old patient with prescribed nitroglycerin who took tadalafil 32 hours ago, complains of chest discomfort and diaphoresis, and has a blood pressure of 150/60 mm Hg Nitroglycerin is a vasodilator that is indicated for patients with cardiac chest pain. Medical direction must be contacted, and the "six rights" must be checked before assisting in the administration of this medication. Nitroglycerin is contraindicated in patients who have a systolic blood pressure below 100 mm Hg, patients with a head injury, patients who have taken erectile dysfunction medications within the previous 48 hours (e.g., Viagra/sildenafil, Cialis/tadalafil, Levitra/vardenafil), and patients who have reached their maximum dose (typically three doses). Most patients are allowed to repeat doses (typically up to three) if the pain remains after five minutes; because of the previously mentioned contraindication, blood pressure should be assessed before administration of initial or repeated nitroglycerin. Carbamazepine (Tegretol) is an anti-epileptic medication and has no effect on nitroglycerin.

In which of the following scenarios is oral glucose most appropriate?

A 58-year-old male who is acting intoxicated and takes injectable insulin A patient with a history of diabetes and an altered mental status may have hypoglycemia. Oral glucose may be given to a conscious patient with suspected hypoglycemia, pending medical direction. The only contraindications to oral glucose are an inability to swallow and unconsciousness, as aspiration can occur. A conscious patient who does not really need glucose will not be harmed by it. However, a patient with a suspected diabetic emergency is a more appropriate scenario for the use of oral glucose. An altered mental status is a common sign of a recent head injury; a patient who has hit their head may exhibit this sign and be treated for a head injury. Trauma patients should not be given anything by mouth. A patient who admits to being intoxicated should not be given anything by mouth due to the risk of vomiting or aspiration.

Which of the following patients would be considered a high priority?

A 62-year-old female with severe chest pain High-priority patients require rapid transport (i.e., spending as little time as needed at the scene). Examples of high-priority patients include those who are unresponsive, are responsive but unable to follow commands, yield a poor general impression, have difficulty breathing, have uncontrolled bleeding, have severe chest pain, have severe pain in any area of the body, and have pale skin/signs of poor perfusion, as well as pregnant patients with likely complicated childbirth.

Which of the following findings are expected for a patient with a concussion injury to the brain?

A patient with a concussion injury will experience a gradually improving mental status, a severe headache, and amnesia of events just before or just after the concussion. Nausea, loss of coordination, and weakness are also common after a concussion. Loss of consciousness is not required to diagnose a concussion, but a brief loss of consciousness is common in most cases. A gradual decline in mental status is not common in concussions and could indicate a cerebral hematoma or another traumatic brain injury.

Which of the following are correct treatments for a patient with pulmonary edema from Congestive Heart Failure (CHF)? Select the three answer options which are correct.

A CHF patient with pulmonary edema can deteriorate rapidly. Treat a patient with Congestive Heart Failure (CHF) and pulmonary edema urgently (following local protocols) and call for ALS (Advanced Life Support) backup. Treatments may include: supplemental oxygen CPAP nitroglycerin position patient to optimize ventilation (sitting in a high Fowler's position) Aspirin would offer no benefit for acute pulmonary edema caused by CHF

Which of the following patients would not be considered a high priority?

A conscious and alert 18-year-old with lower extremity contusions and epistaxis after a bicycle accident High-priority patients require rapid transport. Examples of high-priority patients include those who are unresponsive, are responsive but unable to follow commands, provide a poor general impression, have difficulty breathing, have uncontrolled bleeding, have severe chest pain, have severe pain in any area of the body, have pale skin/signs of poor perfusion, are pregnant with likely complicated childbirth, or experienced a significant mechanism of injury. Significant mechanisms of injury include injury to more than one body system; falls from heights; high-speed motor vehicle, ATV, or motorcycle crashes; car-pedestrian accidents; gunshot wounds; and stabbings. The 18-year-old's injuries are superficial and not life-threatening.

In which of the following situations should patient care be rendered under the criteria of implied consent?

A disoriented 76-year-old patient with no friends or family members present Implied consent is determined if a patient is unconscious or otherwise incapable of making a rational, informed decision. Examples include patients who are intoxicated by drugs or alcohol, mentally impaired, or suffering from a condition that precludes making a rational, informed decision (e.g., head injury). Implied consent only applies when a serious medical condition exists (i.e., a threat to life or limb). Every effort should be made to obtain consent from an available relative prior to treatment, but lifesaving care should not be delayed while waiting for such consent. Expressed consent is the verbal authorization for, or otherwise acknowledgment of, treatment and/or transport. A child is not able to give this consent. For expressed consent to be legitimate, there must also be informed consent. Informed consent is given after the patient understands the risks, benefits, and alternatives to treatment/transport, as well as the consequences of the refusal of treatment. An adult patient must be conscious, alert, and able to make decisions in order to refuse treatment/transport. Ensure the patient has all pertinent information (e.g., assessment, treatment options, consequences of refusal) before you accept a refusal of treatment. A parent or guardian may refuse treatment/transport for a child who does not have life-threatening injuries. Refusal of treatment/transport should be documented and signed by the patient; a witness to the signature, such as a family member or police officer, is appropriate.

A nasopharyngeal airway is contraindicated in which of the following situations?

A patient with ecchymosis behind one ear Nasopharyngeal airways are used for patients with an altered level of consciousness who have an intact gag reflex. Indications for a nasopharyngeal airway include semiconscious or unconscious patients with an intact gag reflex or patients who otherwise will not tolerate an oropharyngeal airway. Contraindications for a nasopharyngeal airway include severe head injury with blood draining from the nose and a history of a fractured nasal bone. A basilar skull fracture, commonly indicated by cerebrospinal fluid draining from the ears, with ecchymosis under the eyes (raccoon eyes) or behind one ear (Battle sign), is an example of a severe head injury. An active gag reflex and consciousness are contraindications to the use of an oropharyngeal airway. A conscious patient who is unable to maintain their airway would likely benefit from a nasopharyngeal airway.

Which of the following patients should not be treated with CPAP?

A patient with facial trauma Contraindications for CPAP (Continuous Positive Airway Pressure) include: respiratory arrest or agonal respirations hypoventilation inability to speak or protect own airway systolic blood pressure < 90 mm Hg chest trauma or signs of pneumothorax active GI bleed or recent GI surgery inability to sit upright inability to tolerate the mask unable to get a good mask seal Respiratory distress, pulmonary edema, and hypoxia are all indications that a patient may need CPAP.

What is the ideal position in which to place a child's head for the delivery of adequate ventilation?

A neutral sniffing position A child's head should be placed in a neutral sniffing position to achieve two goals: keeping the trachea straight and maintaining proper spinal alignment. This position is accomplished by placing the patient on a firm surface, placing a small folded towel under the patient's shoulders and back, and stabilizing the patient's forehead to limit rolling during transport.

Which of the following scenarios would be labeled as the lowest priority in a mass-casualty incident (MCI)?

A responsive 76-year-old female with multiple arm fractures without severe bleeding Immediate-priority patients (red tag) have airway/breathing compromise, uncontrolled/severe bleeding, severe medical conditions, signs of shock, severe burns, or open chest/abdominal injuries. Delayed-priority patients (yellow tag) have burns without airway compromise, major/multiple bone/joint injuries, or back injuries (with or without spinal cord damage). Minimal-priority patients (green tag) have minor fractures or minor soft-tissue injuries. Expectant patients (black tag) are obviously dead, have an obviously nonsurvivable injury (e.g., major open brain trauma), are in respiratory arrest if resources are limited, or are in cardiac arrest.

Which of the following would have rights to otherwise confidential information without expressed patient consent?

A third-party billing representative Any communication between patient and EMS personnel is considered privileged information, even the fact that the patient is under prehospital/hospital care. Additional information that is protected under HIPAA (protected health information, or PHI) includes patient history, assessment findings, and treatment provided. This information may only be disclosed for the purposes of treatment, payment (to third-party billing personnel), or operations, as well as when legally required (e.g., cases of suspected abuse, when subpoenaed). Otherwise, proper authorization—typically documented permission from the patient—is required to disclose confidential information. Personal electronic devices, such as cell phones, should not be used to capture information.

Which of the following can be a common cause of abdominal pain in adult patients?

Abdominal pain can be difficult to diagnose and treat because it occurs in many medical conditions. Some common causes of abdominal pain include appendicitis, gastroesophageal reflux disease (GERD), pancreatitis, gallbladder disease, diverticulitis, and small bowel obstruction. Renal calculi are kidney stones and generally create flank or lower back pain. They are not a common cause of abdominal pain.

When performing the Heimlich maneuver on an adult, where should you deliver the inward and upward thrusts?

Above the umbilicus and below the xiphoid process The abdominal thrust (Heimlich) maneuver is recommended for removing severe airway obstructions in responsive adults and children older than one year. This technique creates an artificial cough by causing a sudden increase in intrathoracic pressure. If a patient is unresponsive, chest compressions should be initiated. Signs of a severe airway obstruction include weak or absent coughing, a decreasing level of consciousness, and cyanosis. Abdominal thrusts should be performed with the rescuer standing behind the patient and wrapping their arms around the patient's body. Place a fist just above the umbilicus and well below the xiphoid process. Then, place the other hand over the fist. Thrusts are performed in an upward direction. If the patient becomes unresponsive, chest compressions, as in CPR, are performed in lieu of abdominal thrusts.

A 14-year-old male was playing baseball and began complaining of left knee pain. Which of the following signs and symptoms would be unexpected with a dislocation?

Absence of pain A dislocation occurs when the bone ends within a joint are no longer in contact. Supporting ligaments are torn, allowing the bone ends to separate. Some dislocations reduce spontaneously. Dislocations of the finger, shoulder, elbow, hip, or knee are the most common. Signs and symptoms include marked deformity, swelling, pain with movement, tenderness on palpation, loss of joint motion, and numbness or impaired circulation to the limb or digit. Dislocations are very painful. The absence of pain may indicate that the dislocation has reduced spontaneously, but most patients still report pain afterward. Some of these signs and symptoms may be seen in alternate conditions (e.g., sprains cause swelling and pain, fractures involve marked deformity and loss of joint motion).

Which of the following findings has the least level of suspicion for a fracture?

Absent peripheral pulses Weak or absent peripheral pulses indicate decreased perfusion to the affected area. This may be due to a cardiovascular condition or shock and not necessarily a fracture. Crepitus (a grating or grinding sensation), false motion (free movement in a limb where no joint exists), and deformity (shortened, rotated, or angulated structures where no joint exists) are often demonstrated by patients with fractures.

A 26-year-old male was involved in a physical altercation with another adult male. Law enforcement advises that the scene is now safe to enter. Upon examination, the patient has an obvious stab wound to his chest that is causing severe pain and distress. A six-inch blade was found by law enforcement at the scene. Which of the following is the most appropriate next step?

Administer high-flow oxygen and cover the wound with an occlusive dressing Penetrating chest injuries may result in air collecting between the lung tissue and the chest wall (pneumothorax). Any penetrating wound in the neck, chest, back, or upper abdomen should be treated with an occlusive dressing to prevent movement of air into the vascular space, thoracic cavity, and/or abdominal cavity. Depending on local protocols, an occlusive dressing may either be taped on three sides to allow air to escape during exhalation or taped on all sides. High-flow oxygen is appropriate for this patient. Because there is no indication that the patient's breathing is inadequate at this time, assisted ventilation is not necessary

A 35-year-old female was recently diagnosed with diabetes. Bystanders stated that her blood sugar was 60 mg/dL approximately ten minutes ago. The patient complains of lightheadedness but denies any other symptoms. Which of the following is the most appropriate next step?

Administer oral glucose Hypoglycemia (low blood glucose level) is often caused by normal insulin dosing with a change in routine (e.g., increased exercise, delayed/skipped/inadequately sized meal), increased insulin use, or an acute illness. The onset is typically rapid. Pale, cool, clammy skin; shallow breathing; hypotension; a rapid, weak pulse; and altered mental status are common. Oral glucose is a key treatment for suspected hypoglycemia. Contraindications to oral glucose use are unconsciousness and an inability to swallow. A normal blood glucose value is between 80 and 120 mg/dL. Since this patient is symptomatic, treatment is appropriate. Oxygen may be beneficial for this patient, but it will not address her hypoglycemia.

A 45-year-old male complains of chest pain. He has a heart condition and took a nitroglycerin tablet about 10 minutes ago; the tablet provided minor relief. After an assessment of his vital signs, which of the following is the appropriate order of interventions?

Administer oxygen as needed to maintain oxygen saturation between 95% and 99%, contact medical control, and request permission to assist the patient in taking another nitroglycerin if his systolic blood pressure is above 100 mm Hg Oxygen should be administered based on the patient's presentation. A nasal cannula will be sufficient for some patients, whereas patients with breathing difficulty may require high flow on a non-rebreather mask. Administer oxygen with a SpO2 oximetry goal of 94-98%. If possible, place the patient in a position of comfort, such as the Fowler or semi-Fowler position. Aspirin may benefit this patient (the correct dose is 324 mg), but medical direction should be obtained prior to interventions that are not part of protocol. As with any medication, inquiries about allergies should be made prior to administration. Some services will use a thrombolytic screen, but these do not take precedence over patient care. His previous positive response to nitroglycerin suggests that a follow-up dose may be warranted; medical direction should be obtained prior to assisting with a repeat dosing. Systolic blood pressure below 100 mm Hg is one of several contraindications to nitroglycerin, so vital signs must be assessed prior to administration. There is no indication that the patient is refusing treatment.

A 52-year-old female patient complains of chest discomfort. She is placed in a comfortable position but appears restless and anxious. The patient's respiratory rate is 28, and her pulse oximetry is 89%. What is the most appropriate next step?

Administer oxygen with a non-rebreather mask at 15 liters per minute A non-rebreather mask should be used on patients with adequate breathing with suspected or confirmed hypoxia. Restlessness, irritability, apprehension, tachycardia, and anxiety are early signs of hypoxia. Late signs of hypoxia include altered mental status, a weak/thready pulse, and cyanosis. Supplemental oxygen via a non-rebreather mask should be delivered at a rate of 10 to 15 liters per minute, providing up to 95% oxygen. A nasal cannula has little use in the prehospital setting. It should be reserved for patients who do not tolerate a non-rebreather mask. Supplemental oxygen via nasal cannula is limited to rates of 1 to 6 liters per minute, providing 24% to 44% oxygen. Assisted ventilation (e.g., a bag-valve mask) is appropriate for patients experiencing severe respiratory distress/failure and inadequate breathing, delivering nearly 100% oxygen.

Which of the following statements about the use of an automated external defibrillator (AED) is correct?

Adult automated external defibrillator (AED) pads can be used on pediatric patients if pediatric pads are not available. An automatic external defibrillator (AED) can be used on a patient of any age and size; it is indicated for any patient who is pulseless and unresponsive. For a pediatric patient, pediatric-sized pads and a dose-attenuating system should be used; if these items are unavailable, adult-sized AED pads can be used.

Which of the following individuals has training in specific aspects of advanced life support, such as intravenous therapy and the administration of certain emergency medications?

Advanced emergency medical technician An advanced emergency medical technician has training in specific aspects of advanced life support, such as intravenous therapy and the administration of certain emergency medications. An emergency medical technician has training in basic life support, such as automated external defibrillation, the use of definitive airway adjuncts, and the assistance of patients with certain medications. An emergency medical responder is the first trained professional (e.g., police, firefighters, lifeguards) who arrives at an incident to provide initial medical assistance. A paramedic has extensive training in advanced life support, such as endotracheal intubation, emergency pharmacology, and cardiac monitoring.

Which of the following is NOT a common time to contact dispatch?

After each treatment/intervention Dispatch is responsible for screening and assigning priority to each call, selecting and alerting the appropriate EMS units, directing EMS units to the scene, coordinating with other public safety services until the incident is over, and providing medical instruction to the caller so that essential care (e.g., CPR) is provided prior to the arrival of EMS. Common times for EMS to contact dispatch include the following: upon receipt of a call (acknowledge and respond to the call), while en route to a call (request directional assistance or additional resources), while on the scene (report arrival, check-in, request additional resources, report leaving the scene), when arriving at the hospital (notify arrival at the point of transfer), and at a return to service (notify that the unit is available for another call). It is not appropriate to contact dispatch after each treatment/intervention. This action would slow patient care and is not required. Depending on local protocols, medical control may require contact before or after specific treatments or responses to treatment

Which of the following is an early sign of respiratory distress in a pediatric patient?

Agitation Early signs of respiratory distress in a child include changes in behavior like combativeness, restlessness or agitation, and anxiety. Additionally, increased work of breathing may be present (e.g., nasal flaring, abnormal breath sounds, accessory muscle use, or the tripod position). Cyanosis, bradycardia, and a decreased level of consciousness are late signs and may occur if respiratory distress is not treated.

Which of the following is the most immediate, life-threatening concern for a patient with a penetrating neck injury?

Air embolism Penetrating injuries to the neck can potentially injure several organ systems. Cervical spine tenderness, subcutaneous emphysema, tracheal deviation, and laryngeal fracture may be discovered by a focused examination of the neck. Wounds that extend through the platysma should not be explored manually; these injuries require evaluation by a surgeon. Subcutaneous emphysema is the presence of air in the soft tissues of the neck. This commonly occurs due to blunt trauma. Subcutaneous emphysema is noted with a characteristic crackling sensation upon palpation of the neck. An air embolism is when air exists within the vascular system. If a vein is punctured within the neck, air can be sucked through to the heart. If enough air is present in the right atrium and right ventricle, cardiac arrest can occur. This would be the most immediate life-threatening concern. If trauma to the neck occurs, a cervical spinal injury is always a possibility. Cervical spine injuries should be immobilized.

A nasal cannula is not appropriate in which of the following scenarios?

All of these Excess oxygen delivery has important consequences for some patients, and hyperoxia can adversely impact outcomes. Therefore, oxygen should only be administered when there is a medical reason. Oxygen in prehospital care is used for treating or preventing hypoxemia, a deficiency in the amount of oxygen reaching tissues. Therefore, only patients with hypoxemia should receive oxygen therapy during prehospital care and only to achieve normal levels. Nasal cannulas are not appropriate if hypoxia is suspected. If hypoxemia is present, a simple or non-rebreather oxygen mask should be used, as its efficiency is considerably greater than that of a nasal cannula. Nasal cannulas are commonly used for patients who do not require high-flow oxygen, cannot tolerate a non-rebreather mask, or have specific illnesses (e.g., COPD, bronchopulmonary dysplasia). Except in rare cases (e.g., a COPD patient with home oxygen therapy), oxygen should only be delivered by a face mask or not at all. If a patient is unable to tolerate anything on their nose or mouth and does not show definitive signs of hypoxia, oxygen can be withheld, as it may cause more discomfort than benefit in these cases.

Which of the following are risk factors for sudden infant death syndrome (SIDS)?

Although it is impossible to predict SIDS definitively, the following are several known risk factors: maternal age younger than 20 years maternal smoking during pregnancy or after birth maternal use of alcohol or illicit drugs during pregnancy or after birth low birth weight Socioeconomic class is not a risk factor for SIDS, as this condition affects all races and socioeconomic classes equally.

Which of the following patients is most likely experiencing cardiac compromise?

An 89-year-old female who had a syncopal episode and has a weak, rapid pulse Chest pain and pressure are common signs of acute coronary syndrome. However, the most common sign of cardiac compromise in a geriatric female patient is weakness. Throat tightness, high fevers, headaches, and gagging are not commonly seen in cardiac compromise. This 43-year-old female is likely suffering from anaphylaxis. This 79-year-old female is likely suffering from an upper respiratory infection. This 63-year-old female could be suffering from a multitude of conditions.

Which of the following scenarios would require the lowest priority in a Mass-Casualty Incident (MCI)?

An unresponsive 64-year-old patient in cardiac arrest with agonal respirations Medical triage allocates care whenever there are more victims than rescuers, when resources are limited, and when time is critical. Immediate-priority patients (red tag) are those with airway/breathing compromise, uncontrolled/severe bleeding, severe medical conditions, signs of shock, severe burns, or open chest/abdominal injuries. Delayed-priority patients (yellow tag) are those with burns without airway compromise, major/multiple bone/joint injuries, or back injuries (with or without spinal cord damage). Minimal-priority patients (green tag) are those with minor fractures or minor soft-tissue injuries. Expectant patients (black tag) are those who are obviously dead, have an obviously non-survivable injury (e.g., major open brain trauma), have respiratory arrest if resources are limited, or are in cardiac arrest.

Which of the following is a way to minimize personal injury during the lifting of a patient?

Avoid twisting of the body while lifting General guidelines for safe lifting of a patient include the following: keeping the legs shoulder-width apart keeping the back in a straight, locked-in position keeping the patient's weight as close to one's body as possible bending at the knees, not the waist, when lifting avoiding lifting and reaching simultaneously avoiding twisting the body while lifting lifting with the palms facing upward communicating constantly with other partners or team members

An unconscious 15-year-old female is being propped up in a sitting position by a bystander at a local pool after striking her head. The patient has a palpable pulse and is breathing. Which of the following is the most appropriate next step?

Apply a cervical collar and provide spinal motion restriction on a long backboard prior to removing her from the pool Patients who fall on their heads, as in diving accidents, will likely have head and/or spinal injuries. These patients require spinal motion restriction to prevent further spinal injury. Spinal motion restriction should include the use of a cervical collar and a long backboard. It is inappropriate to move a patient with a suspected spinal injury unless there is a potential for danger. CPR should be initiated for any patient who does not have a palpable pulse, beginning with chest compressions. Rescue breathing (assisted ventilation without chest compressions) is reserved for patients who have a pulse but are not breathing adequately.

A 59-year-old male with a history of chronic obstructive pulmonary disease (COPD) complains of difficulty breathing that wakes him at night. His skin is hot, and pedal edema is noted. On auscultation, rales can be heard throughout his lungs. Which of the following conditions is a possible cause of this patient's dyspnea?

Any of these Any of the listed conditions is a possible cause of the patient's dyspnea. Causes of dyspnea include pulmonary edema, allergic reaction, pleural effusion, airway obstruction, infection, hyperventilation syndrome, environmental/industrial exposure, carbon monoxide poisoning, and drug overdose. Pneumonia is an infection of the lungs. Risk factors for pneumonia include institutional residency, recent hospitalization, chronic disease processes (e.g., renal failure), immunocompromise, and a history of chronic obstructive pulmonary disease (COPD). Symptoms of pneumonia vary between cases, depending on the cause and severity of the illness as well as the patient's age and overall health. Fevers (i.e., hot skin in this scenario), tachycardia, hypotension (exertional), dyspnea, wheezing/crackles/rhonchi on lung auscultation, dehydration, chest pain, weight loss, and altered mental status are all possible. Congestive heart failure occurs when damaged ventricular heart muscles cannot keep up with the return flow of blood from the atria. Blood tends to back up in the pulmonary veins, causing lung congestion and subsequent pulmonary edema. Tachycardia, hypertension, tachypnea, dyspnea that is alleviated when sitting upright, retractions, rales heard on lung auscultation, chest pain, distended neck veins, peripheral edema, and pale, cyanotic, sweaty skin are all possible. Emphysema, a type of COPD, is a loss of elastic material within the alveolar air space. It is more common than chronic bronchitis, the alternate form of COPD. Adventitious breath sounds (e.g., crackles, rhonchi, wheezing) may be heard on lung auscultation. Dyspnea, chronic coughing, chronic sputum, and long expiration phases are possible in patients with COPD.

Which of the following medical terms are associated with speech? Select the three answer options which are correct.

Aphasia is the loss of the ability to understand or express speech. Agnosia is an inability to speak effectively when asked to name or describe an object. Dysarthria is difficulty speaking caused by brain damage or brain changes from pathological conditions. Aphagia is an inability to swallow

A construction worker has a thin piece of rebar impaled in his abdomen. He is conscious, alert, and complaining of right lower quadrant abdominal pain. A 10-inch piece of rounded metal is protruding from his abdomen. Which of the following should be initiated before moving the patient?

Application of a stabilizing bandage around the rebar Penetrating injuries generally have obvious wounds and external bleeding. Large abdominal wounds may involve protrusion of bowels, fat, or other structures. Pain at the site of the wound, nausea, and vomiting are common. If a penetrating object is still in place, apply a stabilizing bandage around it to control external bleeding and minimize any movement of the object. Inspect the patient's back with anterior penetrating injuries to assess for exit wounds. It is generally contraindicated to try and reduce the size of an impaled object in the pre-hospital setting unless necessary to transport the patient. Trying to reduce the size of an impaled object can cause further injury. The removal of a penetrating object outside of a surgical setting may cause more injury to the area; therefore, in the prehospital setting, penetrating objects should remain in place unless a life-threatening condition exists. The depth of the injury is irrelevant for prehospital care. Visible injuries in this scenario should be treated; internal injuries are concerning but will not be treated until hospital care begins.

Which of the following is not useful in providing spinal motion restriction?

Application of a traction splint A traction splint is useful in the management and stabilization of a femur fracture but does not help with spinal motion restriction. Spinal precautions can be maintained by the application of a rigid cervical collar and securing the patient firmly to the EMS stretcher. They are appropriate for patients who are found to be ambulatory at the scene, patients who must be transported for an extended time (particularly prior to interfacility transfer), or patients for whom a backboard is not otherwise indicated. Patients for whom spinal motion restriction with a backboard is appropriate include those with the following conditions: blunt trauma and altered level of consciousness spinal pain or tenderness neurologic complaint (e.g., numbness or motor weakness) anatomic deformity of the spine high-energy mechanism of injury anddrug or alcohol intoxicationinability to communicatedistracting injury Patients for whom spinal motion restriction with a backboard is not necessary include those with all of the following: normal level of consciousness (GCS 15) no spine tenderness or anatomic abnormality no neurologic findings or complaints no distracting injury no intoxication Patients with penetrating trauma to the head, neck, or torso and no evidence of spinal injury should not be secured to a backboard. Regardless of whether a backboard is used, attention to spinal precautions among at-risk patients is paramount. These include the application of a cervical collar, adequate security to a stretcher, minimal movement/transfers, and maintenance of inline stabilization during any necessary movement or transfers.

A patient is involved in a motor vehicle collision. They complain of neck pain. Inline spinal motion restriction is performed. During the rapid trauma assessment, after an assessment of the neck, what is the most appropriate next step?

Apply a cervical collar The rapid exam is a primary assessment completed after the scene size-up. It includes a general impression, an assessment of the level of consciousness (responsiveness, using the AVPU scale) and an assessment and treatment of life-threatening airway, breathing, and circulatory issues (e.g., major external bleeding). The rapid exam begins at the head in a top-down manner, ending at the back and buttocks. The assessment should be started from the top of the head and then proceed downward, with assessments of the neck, chest, abdomen, pelvis, extremities, and back. In cases where spinal motion restriction is appropriate (e.g., significant trauma with neck/spinal pain, paralysis, neurological deficit, priapism, altered mental status, or intoxication), a cervical collar should be applied immediately following the assessment of the neck. Once the cervical collar is applied, the assessment can continue. As you proceed through the ABCs of life-threatening issues, each finding is treated prior to moving to the next assessment. If a patient is not breathing, your attention focuses on providing ventilatory assistance, even if there is major bleeding. In a trauma case, the priorities are airway, breathing, circulation, neurologic deficit, and environment/exposure.

A 24-year-old male has 36% partial-thickness burns. What is the most appropriate treatment?

Apply a dry, sterile dressing Lifesaving measures for patients with burn injuries include establishing airway control, stopping the burning process, and gaining intravenous access. All clothing and jewelry should be removed from the patient to stop the burning process and to prevent constriction from edema. Partial-thickness burns are painful when air currents pass over the burned surface. Gently covering the burn with dry, sterile dressings relieves the pain and deflects air currents. Do not break blisters or apply an antiseptic agent. Any applied medication must be removed before appropriate antibacterial topical agents can be applied. The application of cold compresses can cause hypothermia. Do not apply cold water to a patient with extensive burns (>10% total burn surface area). A thermal injury is trauma and should be assessed and treated accordingly, following the ABCDEs of trauma care. Immediate lifesaving measures for patients with a burn injury include the recognition of an inhalation injury and the need for endotracheal intubation, the rapid institution of intravenous fluid therapy, and monitoring of the patient's core temperature.

A 29-year-old female has severe bleeding from her forearm. Direct pressure with a pressure dressing has not controlled the bleeding. What is the most appropriate next step?

Apply a tourniquet proximal to the injury Bleeding should be treated with direct pressure over the affected area. If this is unsuccessful, a pressure dressing and subsequent tourniquet above the level of (i.e., proximal to) bleeding may be required to control it. Studies have shown that elevating an extremity with severe bleeding may not be helpful. Do not apply a tourniquet directly over a joint. Never use a wire, rope, or other narrow material that can cut into the skin. Always leave the tourniquet in full view (i.e., do not cover it with a bandage); once applied, do not loosen it unless directed by medical control. Mark the exact time the tourniquet was applied and communicate this information clearly and specifically to hospital personnel upon transfer.

A patient suffered a possible head injury after a fall from a ladder. There is slight bleeding mixed with clear fluid from the ears and bleeding from the back of the skull. In-line spine stabilization and airway management and breathing are completed. Which of the following is not an appropriate treatment option?

Apply pressure to the wound on the back of the skull If an open or compressed skull injury is likely, do not apply pressure to the skull. This patient likely has a basilar fracture based on bleeding mixed with clear fluid (cerebrospinal fluid) from the ears. Alternate signs of a basilar skull fracture include raccoon eyes or Battle's sign (bruising). With all head injuries, continue to assess the patient's level of consciousness and ABCs, prepare for vomiting (to prevent aspiration), and transport the patient promptly.

Which of the following is correct in regard to approaching a medical transport helicopter?

Approach after the pilot signals it is clear to do so Always approach the helicopter in a crouched position from the front and after the pilot has signaled it is clear to do so, regardless of whether the patient is loaded from the side or rear. This approach should be between the 10 o'clock and two o'clock positions. Do not duck under the body, tail boom, or rear sections of the helicopter, as the pilot is unable to see these areas. If the flight crew is present, always follow their instructions.

During a suspected myocardial infarction, which medical intervention inhibits platelet aggregation, thereby preventing clots from forming or enlarging and reducing the overall ischemic damage caused by the infarction?

Aspirin Aspirin reduces fever, pain, and inflammation and inhibits platelet aggregation. It is commonly used for cardiac patients with chest pain. It should only be given with medical direction. Absolute contraindications for aspirin include an allergy and preexisting liver damage; relative contraindications for aspirin include bleeding disorders and asthma. Nitroglycerin is a vasodilator that is indicated for patients with cardiac chest pain. Medical direction must be obtained, and the "six rights" must be checked before assisting in the administration of this medication. Nitroglycerin is contraindicated in patients who have a systolic blood pressure below 100 mm Hg, patients with a head injury, patients who have taken erectile dysfunction medications within the previous 48 hours, and patients who have already taken their maximum dose (typically three doses). Most patients are allowed to repeat doses (typically up to three) if the pain remains after five minutes; because of the previously mentioned contraindication, blood pressure should be assessed before the administration of initial or repeated nitroglycerin. High-flow oxygen is appropriate for patients with chest pain but does not affect platelet aggregation. Diphenhydramine is commonly used for allergic reactions. It is an antihistamine.

As an uncooperative male patient is being transported to the hospital, he threatens to kill EMS personnel after he is released from the hospital. Which of the following has occurred?

Assault Assault is the unlawful placement of a person in fear of immediate bodily harm. This includes threats of violence or restraint and the ability to cause harm. Battery is the unlawful touching of a person. This includes providing emergency care without consent, as well as a physical attack. Defamation is the communication of false information that damages someone's reputation. Libel is the written form of defamation, and slander is the spoken form.

How often should an automated external defibrillator (AED) be checked?

At the start of each shift Equipment, including the automated external defibrillator (AED), should be checked at the beginning of each shift. The battery should be exercised as often as the manufacturer recommends. Additionally, there may be parts of the AED (cables, disposable supplies, etc.) that require checking; this information is commonly provided by the manufacturer in the form of a checklist.

A 73-year-old male is pulseless and apneic. Witnesses say he collapsed 10 minutes ago, and uninterrupted, high-quality bystander CPR was initiated five minutes ago. Which of the following is the most appropriate next step?

Attach an automated external defibrillator and proceed with rhythm analysis and defibrillation The most important therapies for patients suffering from cardiac arrest are prompt cardiac defibrillation and minimally interrupted effective chest compressions. Initiate chest compressions in cases with no bystander chest compressions or take over compressions from bystanders while a second rescuer is setting up the automated external defibrillator (AED) or defibrillator. If adequate, uninterrupted bystander CPR has been performed or if the patient arrests in front of the EMS providers, immediately proceed with rhythm analysis and defibrillation. CPR should be initiated for any patient who does not have a palpable pulse, beginning with chest compressions. High-quality CPR and rapid defibrillation are important factors for survival and should not be delayed. An AED is indicated for any patient who has been assessed to be unresponsive, not breathing, and pulseless. An AED with special pediatric pads is indicated for use on patients between one and seven years old. The opening of the airway and assisted ventilation are completed after the initial 30 chest compressions during CPR. Rescue breathing (assisted ventilation without chest compressions) is reserved for patients who have a pulse but are not breathing adequately.

An unconscious adult patient with a cardiac history has a palpable pulse and is breathing. In this case, management includes all of the following except:

Attaching an automated external defibrillator and analyzing the cardiac rhythm An automated external defibrillator (AED) is contraindicated in a semiconscious or conscious patient or a patient who has a palpable pulse. Do not attach an AED and attempt to analyze the rhythm of a breathing patient with a pulse. An AED is not designed to work as an EKG. This is not a means of management for this scenario. For patients with potential cardiac conditions, high-flow oxygen is an appropriate intervention. This patient is unconscious and cannot be offered other potential interventions for a cardiac issue (e.g., aspirin, nitroglycerin). The focus should remain on the assessment and treatment of ABCs and life-threatening conditions. Additionally, an adequate history may shed light on the current diagnosis. Obtaining a history from family and friends is a great option in this scenario.

Which of the following is true regarding internal cardiac pacemakers?

Automated external defibrillator pads should not be placed directly over a pacemaker. An internal cardiac pacemaker is a device implanted beneath a patient's skin to regulate their heart rate. It typically resembles a small silver dollar under the skin in the upper left chest, under a heavy muscle or a fold of skin. Some pacemakers include an automated implanted cardioverter defibrillator, which monitors the patient's heart rhythm and can slow at an accelerated rate. Automated External Defibrillator (AED) pads or defibrillator paddles should not be placed directly over pacemakers during defibrillation.

Which of the following arrhythmias will an AED shock?

Automated external defibrillators (AEDs) will shock ventricular tachycardia (VTach) or ventricular fibrillation (VFib). It is possible to have Vtach with a pulse, so never place an AED on a patient with a pulse

A 10-year-old girl has a cut on her face. Which of the following is not an appropriate way to interact with this patient?

Bargain with the child if necessary to complete the assessment Use a developmental approach to making the assessment of children. When the patient is a school-aged child (6-12 years old), learn their name and use it while interacting with them. Speak directly to the child and then include the caregiver. Be careful not to offer too much information at once. Anticipate the child's questions and fears and discuss them immediately. Explain in simple terms what is wrong and how it will affect them. Explain procedures immediately before doing them. Thank the child for cooperating. Let the child be involved in their care. Children in this age group are afraid of being out of control. Never lie to a child, telling them that something will not hurt or that you are almost finished if it is not true. Ask older children (like this 10-year-old) if they would like their caregiver to be present. Provide privacy. Do not negotiate unless the child really has a choice. For example, ask if they want the IV line in their right or left hand but do not ask if it is okay to start an IV line if this step is necessary.

An 11-year-old female complains of a possible allergic reaction. Upon arrival, the patient's mother presents an epinephrine auto-injector. Which of the following is the most appropriate next step?

Begin the primary assessment At least a primary assessment must be conducted prior to performing any intervention, including the use of an epinephrine auto-injector. The general impression, formal assessment (e.g., vital signs, DCAP-BTLS), and the determination of the priority of the patient are all involved in the primary assessment. Administering treatment prior to evaluation is negligent. In this scenario, there is no reason to contact a prescribing provider.

Which of the following are components of blood?

Blood is made up of four components: erythrocytes (Red blood cells) leukocytes (White blood cells) platelets plasma Bile is produced in the liver and stored in the gallbladder. It is used in the digestion of food and is not a component of blood.

Which of the following is a late sign of respiratory distress for an adult patient?

Bradycardia Early signs of respiratory distress for an adult patient may include the tripod position, tachypnea, and retractions. As respiratory distress continues and the patient becomes tired from the effort of breathing, hypoxia, bradypnea, bradycardia, and an altered level of consciousness can occur. In severe respiratory distress, wheezing can be audible. Vesicular breath sounds are normal breath sounds.

A patient is experiencing hypovolemic shock due to uncontrolled hemorrhaging. Which of the following signs or symptoms is not expected

Bradycardia Hypovolemic shock is inadequate fluid/volume in the circulatory system. Common causes of hypovolemic shock are bleeding (hemorrhagic shock) and dehydration, such as from inadequate fluid intake, diarrhea, and/or vomiting. Patients will typically present with the following symptoms: anxiety or agitation cool, clammy skin confusion decreased or no urine output general weakness pale skin color (pallor) tachypnea (abnormally rapid breathing) tachycardia (abnormally rapid heart rate)

Which of the following are indications for CPAP administration?

CPAP is indicated for patients experiencing respiratory distress (e.g., from COPD, asthma, or pneumonia). Pulmonary edema is the most common reason for CPAP administration. Apnea is a contraindication to CPAP administration, as the patient must be breathing and able to support their airway for it to be effective.

An adult patient may have alcohol toxicity. Which of the following signs is expected?

Bradypnea Alcohol is a sedative and a hypnotic, so it decreases activity and induces sleep, respectively. In general, alcohol dulls awareness, slows reflexes, and reduces reaction time. Signs of respiratory depression or inadequate breathing and vomiting; in chronic alcohol use, internal bleeding can occur. Severe acute alcohol ingestion may cause hypoglycemia with pale, cool, and moist skin (from sweating); a rapid, weak pulse; potential hypotension and shallow or ineffective breathing; and altered mental status. In most states, patients who are impaired (from mental illness, medical condition, or intoxication) cannot legally refuse transport. Always follow local protocols. Withdrawal from alcohol can cause delirium tremens. This condition commonly involves agitation, restlessness, sweating, tremors, confusion, disorientation, hallucinations or delusions, and seizures. Arrhythmias and hypertension are generally not associated with alcohol use.

Which of the following is not an example of an emergency medical responder?

Bystander trained in BLS Emergency medical responders (EMRs) are individuals within the EMS system who are trained to initiate BLS and other urgent care tasks. These individuals often arrive or are on scene at an incident before an ambulance or emergency medical technicians (EMTs). Examples of EMRs include law enforcement officers, firefighters, park rangers, and ski patrollers. Bystanders are generally not part of the EMS system. These individuals may be labeled "Good Samaritans." Any individual may be helpful if EMS is short-handed; however, sometimes people can interfere with operations and create problems or dangers for themselves and others.

An adult male has sustained a severe facial injury. Which of the following is not a likely cause of an airway obstruction?

Cervical spinal injury Airway obstruction can occur secondary to facial injuries to the face and neck. Clots that are subsequent to severe bleeding, loosened teeth or dentures that become dislodged, and swelling that accompanies an injury can contribute to or cause airway obstruction. Suctioning can be performed for bleeding; however, foreign bodies may require abdominal thrusts, and swelling may require intubation. A cervical spinal injury may affect respirations but will rarely cause an airway obstruction.

The electrical conduction system of the heart controls most aspects of heart rate and enables the four chambers to work together. Which of the following are parts of the heart's electrical conduction system? Select the 3 answer options which are correct.

The heart's conduction system consists of the SA node, the AV node, the bundle branches, and the Purkinje fibers. The circumflex is a branch of the left coronary artery, which supplies blood to the heart muscle. It is not part of the conduction system.

Which of the following directly regulates the blood flow through the capillary beds?

Capillary sphincter Capillary sphincters are circular, muscular walls that constrict and dilate to regulate the blood flow through capillary beds. They are controlled by the autonomic nervous system, which also regulates other involuntary functions such as sweating and digestion. Capillary sphincters respond to outside stimuli: temperature, the need for oxygen, and the need for water removal. Overall, the regulation of blood flow through the capillary beds is determined by cellular needs. The tunica media contains the smooth muscles in the middle layer of the artery. They can contract or dilate to regulate blood flow through the artery and to subsequent capillary beds. Arterioles are the smallest branches of an artery and only exist to connect blood flow from arteries to capillaries; the tunica media can regulate blood flow through the arteries. Arteriovenous shunts allow a bypass of blood past a capillary bed, if a backup of cells happens prior to the capillary. Blood cells flowing through the arteriovenous shunt do not offload their oxygen and may require several passes through these shunts in order to reach the true capillaries.

A 30-year-old patient was electrocuted while working on a high transmission line and has suffered severe electrical burns. Which of the following is a primary concern for this patient?

Cardiac arrest Prehospital care of electrical injury involves three primary steps: remove the patient from contact with the source (using rubber or wood, do not touch the patient directly), assess the need for CPR (ventricular fibrillation is the most common arrhythmia), and transport. An electrical current can cross the chest and cause cardiac arrest or dysrhythmia; cardiac arrest is unlikely to develop if not seen on the initial assessment. Respiratory arrest is also a possible concern when dealing with patients shocked by electricity. Be sure to check for an entrance and exit wound when dealing with electrical burns. Internal bleeding can occur, either immediately or delayed; thrombosis is more common and can result in organ damage. Cardiac arrhythmia is a greater concern in a prehospital setting. Electric shocks have no link to allergic reactions. Heat shock is not a type of shock.

In which of the following conditions does the area around the heart fill with blood or fluid?

Cardiac tamponade Cardiac tamponade is the filling of blood or fluid past the pericardium (the protective membrane around the heart) into the pericardial sac. This most commonly occurs due to penetrating or blunt chest trauma; subsequently, a rupture, tear, or laceration may occur in a coronary artery or vein. Less common causes include cancer and autoimmune diseases (e.g., systemic lupus erythematosus). Due to the increased pressure on the heart, it struggles to fill with blood during diastole. The signs and symptoms of cardiac tamponade are referred to as Beck's triad: bilateral distended jugular veins, narrowing pulse pressure (i.e., a small difference between systolic and diastolic blood pressure), and muffled heart sounds. Altered mental status is common. Oxygen is a mainstay of prehospital treatment. Positive-pressure ventilation should be given to hypoventilating or apneic patients. Angina pectoris occurs when the demand for oxygen by the heart exceeds its supply. An acute myocardial infarction (AMI) is the death of heart muscle due to a lack of blood flow or oxygen. Congestive heart failure occurs when the body can no longer adapt to decreased cardiac output from ventricular heart muscle damage.

Which of the following are considered classes of chemical agents used in Weapons of Mass Destruction (WMD)?

Chemical agents used in WMDs are classified as nerve agents (e.g., VX gas), vesicants (e.g., mustard gas, also known as blistering agents), pulmonary agents (e.g., phosgene, also known as choking agents), and cyanide agents. Viral hemorrhagic fevers are considered biologic agents used in WMDs, not chemical agents. Examples include Ebola and yellow fever.

Which of the following is not a common sign or symptom of congestive heart failure?

Chest pain Congestive heart failure occurs when the ventricular heart muscle is permanently damaged and can no longer keep up with the returning blood from the atria. It can occur after myocardial infarction, heart valve damage, or long-standing hypertension. When the ventricular muscle can no longer contract effectively, the body attempts to maintain cardiac output by increasing heart rate and/or enlarging the left ventricle. Eventually, these adaptations no longer maintain cardiac output, and congestive heart failure develops. Pulmonary and peripheral edema, distended neck veins, hypertension, tachycardia, tachypnea, retractions, cyanosis, diaphoresis, and crackles on auscultation are common; chest pain may or may not appear. Patients with congestive heart failure may find it easier to breathe or achieve alleviation of dyspnea while sitting upright.

An ambulance is responding to a call for an apneic patient. Lights and sirens are used as the ambulance approaches a four-way intersection controlled by traffic lights. The light for the direction of the ambulance's travel begins to change from green to yellow upon approach. Which of the following is the safest way to proceed through the intersection?

Come to a brief stop at the light, look for motorists and pedestrians before proceeding, and move forward when it is safe Intersection crashes are the most common and typically the most serious crashes in which ambulances are involved. If a call is urgent or transportation needs to be rapid, come to a brief stop at an intersection and look for other motorists and pedestrians prior to proceeding through the intersection. Some motorists "time" the traffic lights, which causes an additional hazard. If you are following another vehicle through an intersection, be extra vigilant; other motorists may not be expecting a second vehicle to pass.

Which of the following is expected for a patient who has suffered a retinal detachment?

Complaints of flashes of light Retinal detachment is a medical emergency requiring prompt surgical intervention. In this condition, the retina has pulled away from the choroid (a thin layer of vessels that supply nutrients and oxygen to the retina). Retinal detachment is painless. Early symptoms may include a sudden increase or change in floaters, flashes of light (photopsia), a description of a "curtain" or "veil" falling across the visual field, and blurred vision. Permanent vision loss is possible. Cataracts are the clouding of visual lenses. This condition causes interference with vision, decreased tear production, and difficulty distinguishing colors and seeing clearly. Macular degeneration is the most common cause of irreversible central vision loss in elderly patients. Risk factors include smoking, hypertension, obesity, sun exposure, and a diet low in omega-3 fatty acids and/or dark-green leafy vegetables. Pain when closing the eyelid is a sign of injury to the sclera or cornea of the eye.

Which of the following is a system of internal and external reviews and audits of all aspects of an EMS system?

Continuous quality improvement Continuous quality improvement is a system of internal and external reviews and audits of all aspects of an EMS system. Periodic run-review meetings are held with those involved in patient care to review reports and discuss areas of care that may need changes or improvements. Quality control is the assurance that the appropriate medical care standards are met on each call; this is the responsibility of the medical director. Medical control is the instructions given by a physician via radio or cell phone, as well as standing protocols and guidelines authorized by the medical director of the service program. Primary prevention consists of any effort to prevent an injury or illness from occurring.

Which of the following are considered supraglottic airway devices available for EMS use?

The i-Gel, King LT, CobraPLA, and laryngeal mask airway (LMA) are all considered supraglottic airway devices and are available for EMS use, depending on local protocols. The Shiley is a tracheostomy tube placed in a surgical opening of the trachea, which is known as a stoma. It is not a supraglottic airway available for EMS use.

A 23-year-old male patient was injured in an industrial accident. He is conscious and alert but has a large abdominal laceration with internal organs protruding from the wound. What is the most appropriate treatment for the wound?

Cover the wound with sterile gauze moistened with sterile saline solution and secure with an occlusive dressing An evisceration, an abdominal wound in which organs are protruding outward, should be treated with moistened sterile gauze and secured with an occlusive dressing (taped on all four sides). Due to the ability of an open abdomen to radiate body heat quickly, and because exposed organs will lose fluid rapidly, the organs must be kept warm and moist. Material that is adherent or that loses its substance when wet (e.g., facial/bathroom tissue, paper towel, absorbent cotton) is inappropriate to use. The patient's legs may be flexed to alleviate abdominal pressure if spinal or leg injury is not suspected.

Which of the following are endocrine glands? Select the 3 answer options which are correct.

Endocrine glands of the human body include: adrenal glands pancreas thyroid gland parathyroid glands pituitary gland testes and ovaries The gallbladder is part of the digestive system.

Which of the following suggests a significant mechanism of injury during a motor vehicle collision?

Death of another vehicle occupant If a motor vehicle collision (MVC) occurs, significant mechanisms of injury (MOIs) are suggested by the following findings: death of another vehicle occupant severe deformity of the vehicle or intrusion into the vehicle severe deformities of the frontal part of the vehicle, with or without intrusion into the passenger compartment moderate intrusions from a lateral (T-bone) type of collision severe damage from the rear crashes in which rotation is involved (roll-overs and spins) ejection from the vehicle The involvement of multiple vehicles, the presence of broken glass, or airbag deployment does not necessarily indicate a significant MOI. Airbags can deploy at lower speeds that may not reflect a significant MOI. Airbags decrease the risk of injury to the face, head, and chest, but other serious injuries may happen.

Which of the following are examples of surgically implanted devices that may be used to help patients with cardiac disease?

Devices used in the management of cardiac disease that require surgical implantation include a left ventricular assist device (LVAD), a pacemaker, and an internal defibrillator. The external defibrillator vest does not require surgical implantation and is used as a temporary alternative to a surgically implanted device.

In which phase of the cardiac cycle is the left ventricle at rest?

Diastole Diastole is the phase of the cardiac cycle in which the ventricular muscles rest and the ventricles fill with blood from the corresponding atria. Systole is the phase of the cardiac cycle in which the ventricles contract, pumping blood from the heart into the body via the aorta and into the lungs via the pulmonary artery. Cardiac output is the amount of blood pumped in one minute. It is calculated by multiplying the stroke volume (output of the heart) by the heart rate. Isovolumetric contraction occurs early in systole, which is when the ventricles contract with no corresponding change in volume.

A 27-year-old female is bleeding profusely from a wound on her left leg. There are no signs of skeletal injury. What is the most appropriate means of controlling her bleeding initially?

Direct pressure on the wound Direct pressure on a wound will control most external bleeding and should be attempted first. There is no evidence that elevation of an extremity helps control bleeding. A tourniquet is applied above the level of bleeding if direct pressure is unsuccessful. Tourniquets should be time-stamped, and the receiving hospital advised of their presence and time of application.

A patient's blood pressure is 88/40 mm Hg after the assisted administration of nitroglycerin. What is the most appropriate next step?

Discontinue nitroglycerin and monitor vital signs closely Nitroglycerin is a vasodilator. It relaxes muscles that regulate blood vessel diameter, causing dilation; a subsequent drop in blood pressure is common. Nitroglycerin is contraindicated in patients with a systolic blood pressure below 100 mm Hg. If hypotension occurs due to nitroglycerin use, discontinue it and monitor vital signs closely.

An 87-year-old female is showing early signs of hypoxia. The patient is prescribed continuous oxygen therapy via nasal cannula at a rate of two liters per minute; the oxygen is currently turned off because the patient states she does not like it. Which of the following is the most appropriate next step?

Discuss the appropriateness of supplemental oxygen, place the patient on two liters of oxygen per minute via nasal cannula, and continue the assessment Since the patient is prescribed supplemental oxygen and is currently showing early signs of hypoxia, this treatment is appropriate to resume. Supplemental oxygen should not be delayed for patients showing signs of hypoxia or inadequate breathing. Hypoxia is low oxygenation throughout the body. Early signs of hypoxia in an adult patient include the following: restlessness irritability apprehension anxiety Late signs of hypoxia in an adult patient include the following: altered mental status weak, thready pulse cyanosis dyspnea in conscious patients inability to speak in complete sentences A non-rebreather mask may be an appropriate adjustment in treatment, but the patient's prescribed treatment should be attempted before any changes are made by EMS personnel. There are no indications in this scenario that this patient has inadequate breathing; therefore, assisted ventilation is not appropriate. Assisted ventilation is reserved for patients who are in respiratory distress or respiratory failure. An alert, competent adult patient may refuse treatment; however, risks, benefits, and alternatives to the treatment and its refusal should be explained to the patient prior to accepting a refusal. Any refusal of treatment/transport should be documented and signed by the patient, with a witness.

A seven-year-old male patient jumped from a swing and injured his left leg. There is a prominent deformity of the left leg above his ankle. Which of the following bones was/were most likely fractured?

Distal tibia and fibula The tibia and fibula form the lower leg. Often, both bones are fractured at the same time, even though a single deformity may be noted. Severe deformities should be corrected, and a padded, rigid, long leg splint or air splint (that extends from the foot to the upper thigh) should be applied and secured to the unaffected leg. The femur is the long bone of the upper leg, appearing below the hip and above the knee.

Which of the following is generally not included in a patient care report?

EMS opinion of the incident The following are generally included in a patient care report: patient information and demographics chief complaint level of consciousness vital signs objective and subjective findings from assessments treatment provided times of incident reporting, EMS notification, EMS arrival on scene, EMS departure from scene, EMS arrival at receiving facility, and transfer of patient care Patient care reports should be free of EMS opinion of the incident. Use objective data only.

Which of the following are medications that an EMT may administer if protocols and medical direction allow?

EMTs may administer epinephrine (via an EpiPen), aspirin, and oxygen, depending on local protocols and medical control. An EMT can not administer fentanyl, a narcotic analgesic.

A 27-year-old male with an extensive psychiatric history calmly explains that his thoughts are controlling the whole universe, and he does not know what to do. His body language and speech are gentle and non-threatening. Which of the following responses is appropriate?

Engage calmly and directly In cases of acute psychosis, remain calm and speak to the patient in a respectful, direct, and straightforward manner. Maintain an emotional distance. Explain all procedures and involve people the patient trusts. Do not agree or disagree with the patient's delusions. Do not patronize, argue with the patient, or challenge the validity of their perception. Since this patient is not combative or a threat to others, sedation is not appropriate.

Abdominal pain can be difficult to diagnose and treat because it occurs in many medical conditions. Some common causes of abdominal pain include appendicitis, gastroesophageal reflux disease (GERD), pancreatitis, gallbladder disease, diverticulitis, and small bowel obstruction. Renal calculi are kidney stones and generally create flank or lower back pain. They are not a common cause of abdominal pain.

Ensure no one is touching the patient during analysis and defibrillation No one should touch the patient during analysis and defibrillation. Touching the patient during analysis may alter the AED's ability to assess a cardiac rhythm appropriately. Touching the patient during defibrillation may cause the rescuer to be shocked by the AED. If a shock is advised, chest compressions should be resumed while the AED is charging. Not all AEDs use the anterior/posterior position for pad placement. Studies have shown that the apex/sternal positions have better defibrillation success rates with biphasic shocks. An AED may be used if a patient is in a small puddle of water or in the snow; however, the patient's chest should be dried as much as possible. It is not appropriate to use an AED if a patient is submerged in water.

Which of the following statements is true regarding epiglottitis?

Epiglottitis has a rapid onset of signs and symptoms. Epiglottitis is a life-threatening infection of the soft tissue above the vocal cords. The epiglottis can swell to two to three times its normal size. Bacterial infections are the most common cause. Epiglottitis is more common in infants and children than in adults and has a rapid onset. Patients may look ill, have a very sore throat and high fever, and commonly assume the tripod position to assist with breathing. The airway may be compromised.

Which vessel transports oxygen-rich blood to the lower extremities

Femoral artery The femoral artery is located in the upper leg and carries oxygen-rich blood to the leg. The femoral vein is also located in the upper leg but carries oxygen-poor blood from the legs to the heart. The brachial artery and vein are located in the upper arm and carry oxygen-rich blood to the distal arm and oxygen-poor blood to the heart, respectively.

Which of the following are goals established by the American Heart Association (AHA) for the management of a suspected STEMI (ST-Segment Elevation Myocardial Infarction) patient?

For a patient with STEMI, AHA management guidelines include the following: obtain a 12-lead ECG within 10 minutes of EMS contact with the patient arrange rapid transport to a hospital that offers Percutaneous Coronary Intervention (PCI) begin PCI within 90 minutes from first medical contact to balloon inflation in a cardiac cath lab begin fibrinolytic administration within 30 minutes of the patient's arrival in the ED (Emergency Department) if a cath lab is not available

An adult male trauma patient does not respond verbally or with eye-opening to painful stimuli and speech. He extends his arms and legs during the motor response test. What is his Glasgow Coma Scale (GCS) score?

Four The GCS is an evaluation tool that can determine the level of consciousness. It evaluates eye-opening, verbal response, and motor response. A lower score suggests a more severe extent of brain injury. Survivability is not assessed with GCS but is assessed via the Revised Trauma Score. Eye-opening scoring: spontaneous eye-opening = 4 points eye-opening in response to speech = 3 points eye-opening in response to pain stimuli = 2 points no eye-opening = 1 point Verbal response scoring: responding with an oriented conversation = 5 points responding with a confused conversation = 4 points responding with inappropriate words = 3 points responding with incomprehensible sounds = 2 points not responding verbally = 1 point Motor response scoring: following basic commands = 6 points responding locally to pain = 5 points withdrawing from painful stimuli = 4 points abnormal flexion (decorticate posturing) = 3 points abnormal extension (decerebrate posturing) = 2 points no motor response = 1 point This patient gains a point for not opening his eyes to stimuli (both verbal and pain), a point for no verbal response to verbal stimuli, and 2 points for abnormal extension of his extremities (decerebrate posturing).

What is the most likely cause of dark-colored stool?

Gastrointestinal hemorrhage Gastrointestinal hemorrhage may occur anywhere along the GI tract due to a variety of causes, such as ulcers, esophagitis (gastroesophageal reflux disease), esophageal varices, Mallory-Weiss tears, diverticulitis, gastroenteritis (infection), or hemorrhoids. Melena—dark, tarry stools—commonly occur due to bleeding in the upper gastrointestinal tract. Gallstones commonly cause right upper quadrant or midabdominal pain that may radiate to the right upper back, shoulder, or flank. Appendicitis causes localized right lower quadrant pain, nausea, vomiting, anorexia, fever, chills, and rebound tenderness. Pancreatitis causes upper left and right quadrant pain that radiates to the back and worsens after eating, as well as nausea, vomiting, abdominal distension, and tenderness. Gastrointestinal bleeding is not a common symptom of appendicitis, gallstones, or pancreatitis.

Which of the following are hand tools that may be used in a vehicle rescue?

Hand tools used in vehicle rescues include axes, pry bars, window punches, or striking tools such as sledgehammers. Jaws of life and lifting bags are examples of powered tools that require special training to operate.

A supraglottic device is preferred if intubation attempts fail, but it does not eliminate the risk of vomiting and aspiration as well as an endotracheal tube does. Which of the following are contraindications for using a supraglottic airway such as a King LT or an LMA?

Having an intact gag reflex or known esophageal disease is a contraindication to all supraglottic airway devices. Apnea or failed intubation is an indication for using a supraglottic device such as a King LT (Laryngeal Tube) or an LMA (Aryngeal Mask Airway).

How does an angina pectoris attack differ from an acute myocardial infarction (AMI)?

Heart muscle can be permanently damaged by an AMI. An acute myocardial infarction is a medical emergency, as heart muscle can be permanently damaged within 30 minutes. Angina pectoris is temporary chest pain associated with the heart's need for oxygen that exceeds its supply. Both conditions can cause chest pain/discomfort that is typically described as pressure or heaviness, nausea/vomiting, and sweating. Pain associated with angina pectoris is commonly alleviated with rest and rarely lasts longer than 15 minutes; pain associated with an AMI can last several hours. Pain with both conditions may or may not be alleviated with nitroglycerin.

tient has been stung by a jellyfish while swimming. Which of the following would not be a symptom of envenomation from the sting?

Heart palpitations Envenomation causes painful, red lesions extending in a line from the site of the sting. Some patients may complain of headaches, dizziness, muscle cramps, and fainting. Treatment of a sting from a jellyfish should begin with removing the patient from the water, followed by the scraping of the remaining tentacles with the edge of a stiff object (e.g., a credit card), soaking the affected area in hot water for 30 minutes, and transporting the patient.

Which of the following is responsible for carrying oxygen throughout the body?

Hemoglobin Hemoglobin is the part of a red blood cell (erythrocyte) that carries oxygen, giving red blood cells their coloring. A small percentage of carbon dioxide is carried by hemoglobin. Carbonic acid is a compound that is dissolved in the plasma of the blood. When oxygen is exchanged for carbon dioxide in the body, it is converted into carbonic acid. Platelets are tiny, disc-shaped elements that are smaller than cells and are responsible for the formation of blood clots. Lactic acid is a waste product in the body. It is not involved in oxygen transportation.

A 66-year-old male with a history of chronic hypertension complains of a sudden-onset headache and nausea. The patient reports a strange feeling in his left arm and weakness, numbness, and tingling in his left leg. During the interview, his explanations become confusing, and he complains of visual disturbances. On a physical exam, his pulse is thready at 126 beats per minute, his respiration rate is 20 breaths per minute, and his breathing is not labored. His blood pressure is 218/130 mm Hg. What is most likely occurring?

Hemorrhagic stroke A cerebral vascular accident (CVA), or stroke, should be considered for any patient presenting with an acute neurologic deficit or altered mental status. Common signs and symptoms include the following: sudden loss of motor or sensory control in the body, affecting primarily one side loss of vision, double vision, deficits of the visual field, or nystagmus difficulty swallowing or speaking facial droop sudden difficulty walking or standing sudden alteration of mental status or decreased level of consciousness Ischemic stroke is characterized by the sudden loss of blood circulation to an area of the brain, resulting in a corresponding loss of neurologic function. Acute ischemic stroke is caused by thrombotic or embolic occlusion of a cerebral artery and is the most common presentation, occurring in approximately 85% of CVA patients. In hemorrhagic stroke (15% of patients), bleeding occurs directly into the brain tissue. One common mechanism is likely damage due to chronic hypertension. Patients with intracerebral bleeds are more likely than those with ischemic stroke to have severe headaches, altered mental status, seizures, nausea and vomiting, and/or hypertension. However, none of these findings reliably distinguishes between hemorrhagic and ischemic stroke. Migraine is a complex disorder characterized by recurrent episodes of headaches, most often unilateral and in some cases associated with visual or sensory symptoms, which are collectively known as an aura. Symptoms of migraine include throbbing or pulsatile headaches that intensify with movement or physical activity, nausea (80%) and vomiting (50%, including anorexia and food intolerance), lightheadedness, and sensitivity to light and sound (photophobia). Acute coronary syndrome (ACS) is associated with rupture of an atherosclerotic plaque and partial or complete thrombosis of the infarct-related coronary artery. Although the classic presentation involves substernal chest pain with radiation to the jaw or arms, patients may present with only neck, jaw, ear, arm, or epigastric discomfort. Some patients, including the elderly or those with diabetes, report no pain at all, complaining only of episodic shortness of breath, severe weakness, lightheadedness, sweating, or nausea and vomiting. Elderly persons may also present with only altered mental status. A transient ischemic attack (TIA) is a transient episode of neurologic dysfunction caused by focal brain, spinal cord, or retinal ischemia without acute infarction. TIAs are often labeled "mini-strokes" due to their relatively benign immediate consequences. However, TIA can indicate the likelihood of a coming stroke; therefore, the term "warning stroke" is more appropriate. Approximately 33% of people who have a TIA have a more severe stroke within the following year.

An alert and oriented patient may be suffering from carbon monoxide poisoning. Which of the following is the best treatment for this patient?

High-flow oxygen via non-rebreather mask Carbon monoxide is a colorless, odorless, tasteless, and highly poisonous gas. It is the leading cause of accidental poisoning deaths in the United States. Permanent brain damage can occur. Patients may initially believe they have the flu; common signs and symptoms include headaches, dizziness, fatigue, nausea, vomiting, dyspnea on exertion, chest pain, impaired judgment, confusion, or even hallucinations. If more than one patient in the same environment presents with these signs, carbon monoxide poisoning should be suspected. Once patients have been removed from the environment, their condition should improve. High-flow oxygen by a non-rebreather mask is the best treatment for conscious patients. Patients who are unconscious or have an altered level of consciousness may need an airway adjunct and assisted ventilation via a bag-valve mask. In severe cases, hyperbaric or pressurized oxygen therapy may be required.

An adult patient has widespread burns from an explosion. He has superficial, partial, and full-thickness burns over a large portion of his body. Which of the following is the most appropriate treatment regimen?

High-flow oxygen, dry sterile dressings, treatment for shock, and maintenance of body temperature The first responsibility in caring for burn patients is to stop the burning process. The patient should be moved from the burning area and wrapped in a fire blanket if available. Smoldering clothing and/or jewelry should be removed if possible. Do not pull at clothing or accessories if resistance is felt, as this may further exacerbate damage to the tissue. If the burn covers under 10% of the body, the burned area may be immersed in cool water or saline solution if allowed by local protocols. High-flow oxygen should be given to patients with large burn areas, as the inhalation of irritants is likely. Cover the burns with a dry sterile dressing to prevent further contamination. Do not use ointments, lotions, or antiseptics. Do not intentionally break blisters. Treat any life-threatening conditions as they are found. Treat for shock and cover the patient in warm blankets to prevent heat loss. Transport rapidly.

Which of the following signs or symptoms is most likely to be associated with a heat-related emergency?

Hot, dry skin Heat exhaustion is the most common heat emergency. Hypovolemia, as a result of loss of water and electrolytes from heavy sweating, is a common cause. Typical signs and symptoms include dizziness, weakness, altered mental status/syncope, nausea/vomiting, headaches, muscle/abdominal cramping, dry tongue, and thirst. A slightly elevated temperature, a rapid and weak pulse, and low diastolic blood pressure are possible. Skin changes are likely, but they depend on the level of exposure. Moist, pale, cool skin indicates excessive fluid and salt loss. Hot, moist skin and hot, dry skin indicate the body's inability to regulate core temperature.

A 32-year-old male patient appears to be intoxicated. Bystanders report that the patient seemed to be fine until he suddenly began acting drunk. His pupils are normal, and his skin is pale, cool and clammy. Which of the following is the most likely cause of these symptoms?

Hypoglycemia Hypoglycemia, hyperglycemia (diabetic ketoacidosis, or DKA), and alcohol intoxication may all present with altered mental status. Hypoglycemia is more likely than intoxication to present with a rapid onset, as in this patient with no history or report of alcohol use. Hypoglycemia frequently causes clammy skin. If a fruity odor of ketones on the breath is evident, suspect DKA. Opioid use would result in small or pinpoint pupils, which this patient does not demonstrate. An allergic reaction will commonly involve hives, a rash, urticaria, respiratory complaints, and/or swelling.

Which of the following is NOT true regarding interventions for fractures?

If bone ends are protruding, attempt to push them back into place.** DO NOT Never attempt to push the end of a protruding bone back into place. This action increases the risk of infection and the possibility of further damage. A locked joint is difficult to move; it is often the sign of an underlying fracture. If a joint is locked, do not try to manipulate the limb excessively to elicit pain with motion. Splinting can protect and maintain the position of an injured extremity while inhibiting motion. Splinting helps prevent further damage from broken bone ends, restriction of distal blood flow due to the pressure of bone ends on blood vessels, excessive bleeding at the injury site, increased pain, and paralysis resulting from a damaged spine. Neurovascular status—including pulse, sensation, and movement—should be assessed before and after splinting. Most splinting is done in a position of comfort; splinting of the wrist and hand should be done in a position of function.

A 55-year-old male has an extensive cardiac history and may be experiencing cardiac compromise. When is the most appropriate time to attach automated external defibrillator (AED) pads?

If he becomes pulseless and apneic An automated external defibrillator (AED) is indicated for use on unresponsive and pulseless patients. Defibrillation is not a substitute for high-quality CPR but is another factor that promotes survival. The deterioration of the patient's condition (excluding the absence of a palpable pulse), the ineffectiveness of nitroglycerin use, and any part of a patient's history are neither indications for nor contraindications to the use of an AED.

Which of the following are unpredictable situations that may occur while responding to an emergency call? Select the two answer options which are correct.

It is important to become familiar with traffic patterns and roads in your response area in order to plan your response routes to emergency calls or transport routes to hospitals. Heavy commuter traffic times and railroad crossings can be predicted based on known traffic patterns and train schedules in the response area. It is recommended to avoid rail crossings whenever possible due to the sometimes unpredictable stalls or blockages. It is difficult to plan response routes based on inclement weather or stalled vehicles because these are unpredictable situations.

A five-year-old is unconscious and breathing inadequately. When an oropharyngeal airway is inserted, which of the following is true?

If the airway is too large, it may obstruct the larynx. An oropharyngeal keeps the tongue from blocking the airway and allows for easier suctioning. An oropharyngeal airway is indicated for patients who are unconscious and do not have an intact gag reflex. Oropharyngeal airways are contraindicated in patients who may have ingested caustic or petroleum-based products, as the airway may induce vomiting. Measure an appropriately sized airway for children by placing the flange at the central incisor, with the bite-block segment parallel to the hard palate; the tip of the airway should reach the angle of the jaw. Alternatively, length-based resuscitation tape (Broselow tape) may be used to size an airway. After choosing an appropriately sized oropharyngeal airway, position the patient's airway via the head-tilt/chin-lift or jaw-thrust maneuver, depending on the scenario. For children, avoid hyperextension of the neck. Depress the tongue with either a tongue blade or the airway tip and slowly insert the airway. Rotate the airway into position, following the curve of the tongue. The flange should rest against the lips. Reassess the airway after it is in place. If the airway is too small, the tongue may obstruct the airway; if the airway is too large, the airway may obstruct the larynx.

An adult patient suddenly develops respiratory distress after a bee sting. Under medical direction, when can epinephrine be administered?

If the medication has been prescribed for this patient by their physician An epinephrine auto-injector can be used for a severe allergic reaction. Medical direction is required. Ensure the medication has been prescribed to the patient on whom it is to be used and verify the medication has not expired. The patient does not need to be in severe respiratory distress/arrest to receive an epinephrine auto-injection. The presence or absence of heart disease does not affect the indication for an epinephrine auto-injector. EMTs can administer epinephrine under medical direction only if the medication was previously prescribed for that patient and they are experiencing the adverse signs and symptoms of a severe allergic reaction, which affects the respiratory system. It would not be appropriate to allow a patient to suffer and experience unneeded distress when a medication is available. Medical control or local protocols do not make EMTs wait until a patient is in full arrest to intervene when the prescribed medication could have prevented the arrest. A history of heart disease is not a contraindication to an EMT giving a previously prescribed medication for an allergic reaction when medical control orders have been issued. The medication does not have to be stored in a refrigerator before use.

After experiencing general malaise, a 52-year-old male suddenly collapses and quickly becomes apneic and pulseless as the ambulance arrives at the hospital. Which of the following is the most appropriate next step?

Initiate CPR, beginning with chest compressions, and attach an automated external defibrillator (AED) if available CPR should be initiated for any patient who does not have a palpable pulse. High-quality CPR and rapid defibrillation are important aspects of survival. However, CPR should not be delayed to make other preparations for entry into an emergency department. This patient will require assisted ventilation, as he is apneic; however, chest compressions are completed prior to ventilations in CPR.

Which of the following is a sign of increased pressure within the chest, secondary to a cardiac tamponade?

Jugular vein distention Increased chest pressure may occur due to trauma, pneumothorax, hemothorax, or constrictive pericarditis and cardiac tamponade. Jugular vein distention, hypotension, and collapse of the lung may occur if chest pressure is increased. A palpable carotid pulse is a normal finding. Hypertension is not a sign of increased chest pressure. Increased chest pressure leads to hypotension and the inability of the lungs to expand fully.

A 72-year-old male states that his wife recently died in a car accident. He is depressed and speaks of wanting to join his wife in heaven. During history taking, many pills are noted in a bowl on his dining table. Which of the following is the greatest concern?

Intentional overdose Risk factors for depression include a history of depression, chronic disease, and loss of function, independence, or a loved one. For a patient who is exhibiting signs of depression, it is appropriate to ask whether they are considering suicide. If the answer is "yes," the next question should be, "Do you have a plan?" All information should be included in the documentation. Older men have the highest successful suicide rate of any age group in the United States. White men 85 years or older are at the highest risk and typically use firearms as their suicide method of choice. Since this patient is exhibiting signs of depression, opioid addiction is possible, but suicide is a greater concern.

A 20-year-old male has a suspected closed head injury and a closed femur fracture. He feels cool and clammy and appears anxious. The patient's respirations are 22 breaths per minute, his heart rate is 120 beats per minute, and his blood pressure is 100/48 mm Hg. Which of the following is the most likely cause of shock in this case?

Internal bleeding around the femur fracture Hypovolemic shock, which is due to an inadequate amount of fluid or volume in the circulatory system, can be caused by bleeding (hemorrhagic) or dehydration (non-hemorrhagic). Common signs and symptoms include an altered mental status; a rapid, weak pulse; hypotension; tachypnea; cyanosis; and cool, clammy skin. Since this patient has a closed femur fracture, he has the potential to lose up to one liter of blood. An open femur fracture may result in the loss of even more blood. In either event, hypovolemic shock is not unusual. A closed head injury is not likely a cause of shock, but a damaged cervical spine may cause neurogenic shock. Severe pain may cause psychogenic shock, but this is less likely than hypovolemic shock due to internal bleeding. A disrupted aorta would result in signs and symptoms of shock, but there are no signs of aortic disruption in this scenario (i.e., chest or lower back pain, unequal pulses in extremities, pulsatile mass in the abdomen).

Which of the following types of motor vehicle accidents involving an ambulance is most common?

Intersection crashes The majority of accidents involving ambulances occur at intersections. These accidents are usually the most serious as well. It is important to come to a brief stop at a red light and/or an intersection and look for other motorists and pedestrians before proceeding. Often, other motorists attempt to "time the traffic lights," provoking a serious crash. Additionally, when following another emergency vehicle, do not assume that the intersection is clear since other motorists may not be expecting a second vehicle. Hydroplaning, failing to stop, and crashing into other emergency vehicles are not as common as intersection crashes.

Gas exchange in the lungs and tissues is known as respiration. Which of the following are considered external factors that may affect this process?

Low-atmospheric-pressure conditions (e.g., at high altitudes and with carbon monoxide inhalation) are considered external factors that will affect respiration. COPD (Chronic Obstructive Pulmonary Disease) and pulmonary edema are considered internal factors that can affect respiration.

Which of the following is true regarding the proper lifting technique?

Keep the patient as close to your body as possible When lifting, the legs should be kept approximately 15 inches (38 cm) apart (i.e., shoulder width). The rescuer's center of gravity should be balanced between the feet. The back is held upright, and the upper body is brought down by bending the legs. The patient or stretcher is grasped and lifted by straightening the legs until a standing position is achieved while keeping the patient as close to the body as possible. Your arms should be curled to waist height and approximately shoulder distance apart. Ensure that your arms are not outstretched when lifting. Avoid placing lateral force across the spine.

A 19-year-old male presents with costovertebral angle tenderness with guarding. He is conscious with a respiration rate of 16 breaths per minute, a pulse rate of 108 beats per minute, and a blood pressure of 138/80 mm Hg. He denies any injury and states that the pain began this morning when he awoke. Which of the following organs is most likely involved?

Kidneys The kidneys are approximately five inches (13 centimeters) long and appear just anterior to the costovertebral angle (the area between the ribs and the spine). Tenderness in this area is common in patients with kidney insult. Hematuria may also occur. If blood loss is significant, the patient may show signs of shock. Due to the uncertainty of diagnosis in patients with hematuria and flank pain, rapid transportation is appropriate. The liver is in the upper right quadrant of the abdomen and rarely causes costovertebral angle pain. The head of the pancreas connects with the duodenum slightly to the right of the midline. It is most likely to cause epigastric pain, radiating to the back. The gallbladder is a pear-shaped, hollow structure located under the liver and on the right side of the abdomen. It stores and concentrates bile, a digestive enzyme produced by the liver. It is less likely to cause costovertebral angle pain.

Which of the following patients have a significant mechanism of injury (MOI) and should be transported to the highest-level trauma center available? Select the 2 answer options which are correct

MOIs that are considered significant and require transport to the highest-level trauma center available include: Falls Adults: > 20 feet (one story = 10 feet) Children: > 10 feet or two to three times the height of the child High-risk auto crashes Intrusion, including roof: > 12 inches occupant site; > 18 inches any site Ejection (partial or complete) from automobile Death in the same passenger compartment Vehicle telemetry data consistent with a high risk of injury Automobile versus pedestrian or bicyclist thrown, run over, or with significant (> 20 mph) impact Motorcycle crashes > 20 mph

An adult male who took a drug overdose in a suicide attempt, per 911 dispatch, is unconscious with shallow breathing and a slow pulse. He is exhibiting constricted pupils and cyanosis. Which of the following drug types is the most likely cause of this patient's overdose?

Narcotic Recognition of the agent behind an overdose is important, as signs and symptoms may help differentiate possible causes. Opiate/opioid or narcotic (e.g., morphine, codeine, oxycodone, methadone, heroin) overdoses typically present with hypoventilation/respiratory arrest, hypotension, pinpoint/constricted pupils, and sedation/coma, as in this scenario. The patient's cyanosis is likely secondary to hypoventilation. Anticholinergic and antihistamine (e.g., diphenhydramine, atropine, chlorpheniramine, doxylamine) overdoses will cause tachycardia, hypertension, hyperthermia, dilated pupils, dry skin/mucous membranes, decreased bowel sounds, and sedation/coma. Sympathomimetic (e.g., epinephrine, albuterol, cocaine, methamphetamine) overdose will cause tachycardia, hypertension, hyperthermia, dilated pupils, and agitation or seizures. Benzodiazepine (e.g., diazepam, secobarbital, flunitrazepam, midazolam) overdose will cause hypoventilation, hypotension, slurred speech, and sedation/coma. Sedative hypnotics generally do not affect pupil diameter.

For which of the following wounds would an occlusive dressing be indicated?

Occlusive dressings are used to keep air or fluid from entering the wound. Examples include penetrating wounds to the trachea, chest, mediastinum, or great vessels of the neck. Burn injuries should be covered with dry, sterile dressings that are non-adherent if possible, not an occlusive dressing. An avulsion to the finger would not be managed with an occlusive dressing but with sterile gauze and dressings.

An appropriately trained EMT did not provide adequate ventilation during resuscitation, and a patient died as a result. Which of the following has occurred?

Negligence Negligence is the failure to provide the care that a healthcare professional with similar training would provide in the same or a similar situation. Determination of negligence is based on the duty of the provider, the breach of said duty, the physiological or psychological harm/damage done, and the causation of intervention or lack thereof in that damage. All four elements must be present for negligence to apply. Defamation is the communication of false information in writing (libel) or speech (slander) that damages a person's reputation. Battery is the unlawful touching of a person, including providing emergency care without consent. Malpractice is used to describe unethical or harmful practices by a medical doctor or practitioner.

A 56-year-old male patient complains of chest pain. He has a history of angina pectoris. Which of the following medications is most likely to alleviate his pain, if it is cardiac-related?

Nitroglycerin Nitroglycerin is a vasodilator that is indicated for patients with cardiac chest pain. Medical direction must be contacted, and the "six rights" must be checked before assisting in the administration of this medication. Nitroglycerin is contraindicated in patients who have a systolic blood pressure below 100 mm Hg, patients with marked tachycardia, patients with a head injury, patients who have taken erectile dysfunction medications within the previous 48 hours, and patients who have already taken their maximum dose (typically three doses). Most patients are allowed to repeat doses (typically up to three) if the pain remains after five minutes; because of the previously mentioned contraindication, blood pressure should be assessed before the administration of initial or repeat nitroglycerin. Aspirin has many properties, including the inhibition of platelet aggregation. Aspirin may reduce the likelihood of an acute myocardial infarction, if one has not occurred, and may prevent any active clots from increasing in size. Aspirin will not directly help this patient's chest pain, but it reduces mortality. If a patient, especially one with a history of angina, is having chest pain, nitroglycerin should be given to decrease the pain. It dilates the coronary arteries and improves blood flow through them. Acetaminophen has some antipyretic and anti-inflammatory properties but is not indicated for cardiac chest pain. Furosemide is a diuretic that is often used for congestive heart failure.

An unresponsive 40-year-old male is apneic and cyanotic. He has a weak carotid pulse of 48 beats per minute. Rescue breathing via a bag-valve mask with a reservoir and supplemental oxygen is initiated. What is the appropriate ventilation rate for this patient?

One breath every five seconds to mimic a normal respiration rate A patient in respiratory distress or failure requires artificial ventilation. A bag-valve mask is an adequate method of artificial ventilation and can deliver up to 100% oxygen flowing at 15 liters per minute and a reservoir bag. Ventilation should be given to an adult patient every five seconds and to a child patient every three seconds; these rates mimic a normal ventilation rate. Any rate below these values may not provide adequate oxygenation. Ventilation should cause the chest to rise and fall and avoid gastric distension. Gastric distension may lead to vomiting and subsequent aspiration. Rapid ventilation should be avoided due to the risk of gastric distension.

An adult trauma patient is unresponsive but breathing at a rate of 12 breaths per minute with snoring respiration. Which of the following is the most appropriate intervention?

Open the airway using the jaw-thrust maneuver while maintaining in-line cervical spine stabilization and apply an oropharyngeal airway The jaw-thrust maneuver is the appropriate method of opening the airway for a patient with suspected cervical spinal trauma. If the mechanism of injury is unknown, as in this case, suspect a possible cervical spine injury. In-line stabilization of the cervical spine is used to ensure no further damage occurs until a cervical collar can be applied. To perform the jaw-thrust maneuver, the rescuer kneels above the patient's head with their fingers behind the angles of the lower jaw. The jaw is then moved upward with the index and middle fingers while the thumbs help position the lower jaw. The head-tilt/chin-lift maneuver is used to open the airway of patients who do not have suspected spinal trauma. An oropharyngeal airway keeps the tongue from obstructing the airway and eases suctioning, if necessary. It is contraindicated in patients who are conscious or have an active gag reflex. A nasopharyngeal airway is generally reserved for patients who have an active gag reflex and cannot maintain a patient's airway spontaneously. It is contraindicated in patients with severe trauma to the head or face or a history of a fractured nasal bone. Assisted ventilation is appropriate for patients who are in respiratory distress or arrest.

Which of the following is an appropriate intervention during the triage of a mass-casualty incident?

Opening the airway of an unconscious patient with a pulse Simple assessment and correction of life-threatening conditions may be completed during triage (e.g., opening the airway, control of severe bleeding). Environmental conditions and fractures are not treated during triage but may affect the priority of a patient. A pulseless patient is considered expectant (black tag) during a mass casualty incident; these patients are given the lowest priority, as they are not likely to survive.

A 19-year-old female is complaining of general malaise and shortness of breath. She is sweaty, is breathing very shallowly, and has pinpoint pupils. Which of the following is the most likely cause of her signs and symptoms?

Opioid overdose Opioid overdose typically presents with sedation/unconsciousness, bradypnea, cyanosis, and pinpoint pupils. As respirations continue to decrease in opioid overdose, respiratory and subsequent cardiac arrest are possible. Adequate airway and ventilation are paramount. Naloxone is an antidote that reverses the effects of opioid overdose; it is given intravenously, intramuscularly, or intranasally. As naloxone begins to work, the patient may regain consciousness and will no longer be able to tolerate an oropharyngeal airway if used. Sedative-hypnotic drugs (e.g., barbiturates, benzodiazepines, muscle relaxers, lithium) are sedating substances. Patients develop slurred speech, sedation or coma, hypoventilation, and hypotension. Sympathomimetics (e.g., cocaine, MDMA, PCP, amphetamine/methamphetamine) are central nervous system stimulants that mimic the sympathetic nervous system. Common signs and symptoms of use include tachycardia, hypertension, and dilated pupils. A tension pneumothorax is caused by damage to the lung tissue. The damage allows air normally held within the lung to escape into the chest cavity. The lung subsequently collapses, and air can begin to apply pressure to the structures in the mediastinum (e.g., heart, great vessels). The patient will present with a trachea that deviates toward the unaffected side, along with tachycardia, tachypnea, dyspnea, anxiety, and decreased breath sounds. The onset of a tension pneumothorax is slower than that of a spontaneous pneumothorax.

Which of the following are common over-the-counter (OTC) medications?

Over-the-counter (OTC) medication can be purchased without a prescription. Aspirin, acetaminophen (Tylenol), diphenhydramine (Benadryl), and ibuprofen (Advil, Motrin) are common OTC medications. Prescription medications cannot be purchased without a prescription from a healthcare provider. Ipratropium (Atrovent) is a prescription medication.

Overaggressive ventilation with a bag-valve mask device can complicate a respiratory failure situation and can cause which of the following? Select the 3 answer options which are correct.

Overaggressive ventilation with a bag-valve mask device can cause barotrauma from too much tidal volume or excessive pressure in the airway. Overaggressive ventilation with a bag-valve mask device also causes gastric distension, which can lead to vomiting and aspiration. Hypocapnia, an excessive loss of carbon dioxide, is also common in overaggressive ventilation due to high minute volume. Too much carbon dioxide (hypercapnia) is common in an airway obstruction or a low respiratory drive such as narcotic overdose, and it is not a result of overaggressive ventilation.

Which of the following oxygen-delivery devices and concentrations provided are listed accurately?

Oxygen concentrations vary based on the flow rate selected. The higher the flow rate is, the greater the O2 concentration is delivered. Generally, a nasal cannula delivers 24-44% oxygen. A non-rebreather mask delivers up to 90% oxygen. A bag-valve mask with an oxygen reservoir can deliver nearly 100% oxygen. A venturi mask can deliver 24%, 28%, 35%, or 40% oxygen, depending on the adapter used. There is not an adapter that will allow a venturi mask to deliver 100% oxygen.

Which of the following interventions should be implemented first as soon as hypovolemic shock is suspected?

Oxygen supplementation and assisted ventilation if needed Shock is a life-threatening condition, and the treatment follows trauma protocols. On-scene assessment and care should consist of ABCs with spinal precautions, a rapid trauma exam, immobilization, and transport. Provide airway support with supplemental oxygen and assisted ventilation if breathing is inadequate, then control obvious external bleeding to preserve circulation (life-threatening bleeding may be addressed prior to airway/breathing). Next, implement spinal immobilization, temperature management (blankets), and transport to a trauma center per protocols. Do not provide anything by mouth in a prehospital setting. Vital signs are assessed every five minutes for a patient in shock. Secondary assessments, treatments, and individual extremity fractures should be splinted en route.

Which of the following are criteria that must be met to withhold using a long spine board?

Patients who do not require a backboard include those with: a normal level of consciousness (Glasgow Coma Score 15) no spine tenderness or anatomic abnormality no neurologic findings or complaints no distracting injury no intoxication Always follow local protocols. Patients with numbness in their hands or fingers or other neurological complaints should receive spinal motion restriction with a spine board.

Which of the following is not an appropriate treatment for hypothermia?

Place hot water bottles directly on the skin of the axilla and groin Passive rewarming is an appropriate first step when treating a patient who has mild hypothermia. Passive rewarming includes: placing the patient in a warm environment (e.g., the patient compartment of the ambulance with the heat elevated) removing wet clothing applying heat packs/hot water bottles to the groin, axillary, and cervical regions placing dry blankets over and under the patient giving warm, humidified oxygen Heat packs/hot water bottles should not be applied directly to the skin to avoid burns. Stimulants such as coffee, tea, soda, or tobacco products should be avoided, as they are vasoconstrictors, which may further impair circulation. Patients should be discouraged from walking to spare their feet from damage. Massaging the extremities or handling the patient roughly is inappropriate.

A trauma patient has sustained several fractures. Which of the following fractures would be considered the most critical injury?

Pelvic fracture A pelvic fracture is a life-threatening condition. It is often the result of direct compression in the form of a heavy blow that crushes the pelvis. However, not all pelvic fractures are due to violent trauma; an older patient with osteoporosis may suffer from a pelvic fracture due to a simple fall. Several liters of blood may drain into the pelvic space and retroperitoneal space due to a pelvic fracture. Hypotension, shock, and death may occur. Often, there are no visible signs of bleeding until severe blood loss has occurred. Tenderness is the most reliable sign in the setting of a pelvic fracture. Fine compression of the iliac crests will produce pain at the fracture site. Stabilization using a pelvic binder, backboard, or scoop stretcher is appropriate. A femoral shaft fracture can create a blood loss of 500 to 1000 mL in a closed fracture and even more in an open fracture. Hypovolemic shock can develop. While a femoral fracture is a high-priority condition, it is not as serious as a pelvic fracture. Humeral and tibial fractures are not commonly life-threatening.

What is the fibrous sac around the heart called

Pericardium The pericardium is the fibrous sac that surrounds the heart. If this sac fills with blood or fluid, cardiac tamponade occurs. The myocardium is the muscle tissue of the heart. The membrane that lines the abdominal cavity is the peritoneum. The mediastinum is a space between the lungs that contains the heart, great vessels, esophagus, trachea, major bronchi, and many nerves.

An infant receives a 1 for skin color during an Apgar assessment. Which of the following signs must be evident?

Peripheral cyanosis Appearance is assessed during the Apgar scoring of an infant at 1 and 5 minutes after birth. If the newborn is entirely blue or pale, the score is 0. If the newborn has a pink body, but their hands and feet remain blue (peripheral cyanosis), the score is 1. If the entire newborn is pink, their score is 2. Cardiac insufficiency, neonatal sepsis, and central pontine demyelination are not evaluated by Apgar scoring

An 18-year-old patient is showing signs and symptoms of amphetamine use. Which of the following symptoms is NOT expected?

Pinpoint pupils Sympathomimetics (e.g., cocaine, amphetamine/methamphetamine, epinephrine, albuterol) are stimulants that mimic the effects of the sympathetic nervous system. Common signs and symptoms of use/overdose include hypertension, tachycardia, dilated pupils, agitation, seizures, and hyperthermia. It should be noted that anticholinergics (e.g., atropine, diphenhydramine) have similar signs and symptoms. Pinpoint pupils are commonly seen with opiate use, not with sympathomimetic use.

A 20-year-old patient has sustained a hand injury. During splinting of the patient's hand, how can a position of function be maintained?

Place a soft roller bandage on the palm of the hand For splinting of an injury to the wrist or hand, the limb should be supported, and the hand should be placed into the position of function (i.e., the wrist slightly bent down and all finger joints moderately flexed). This may be completed by placing a soft roller bandage on the palm. Alternate positioning of the hand is inappropriate.

Which of the following measures is appropriate emergency care for an adult with an early, localized cold injury?

Protecting the affected body parts from further cold exposure A patient with a localized cold injury should be removed from the area to prevent further exposure. The affected area should be handled gently, and any wet or restrictive clothing should be removed to avoid further injury. The area should be splinted and covered with a dry, sterile dressing. Rubbing affected areas is potentially harmful and should be avoided. Never place heating pads directly on a localized cold injury. Never attempt active rewarming in hot water of an extremity pre-hospital. This is only done in the emergency department under direct physician observation.

What is the correct technique for performing the head-tilt/chin-lift procedure on a patient with no suspected spinal trauma?

Place the heel of one hand on the patient's forehead and apply firm backward pressure. Place the fingertips of the alternate hand under the lower jaw and lift the chin upward. The head-tilt/chin-lift maneuver is completed with the patient in a supine position and the rescuer beside the patient's head. The heel of one hand is placed on the patient's forehead, and firm pressure is applied to tilt the patient's head back. The fingertips of the alternate hand are placed under the lower jaw, near the bony part of the chin; the chin is then lifted upward. The forehead is held to maintain the backward tilt. If spinal trauma is suspected, the jaw-thrust maneuver is performed. Inline stabilization of the cervical spine is used to ensure no further damage occurs until a C-collar can be applied. The rescuer kneels above the patient's head with their fingers behind the angles of the lower jaw. The jaw is then moved upward with the index and middle fingers, and the thumbs help position the lower jaw.

A 58-year-old female is complaining of chest pressure and experiencing difficulty breathing. Her skin is cool and pale. Her SpO2 is 96% on room air. She states the pain began an hour prior to EMS arrival. Which of the following is the most appropriate next step?

Place the patient in a position of comfort The treatment of chest pain begins with placing the patient in a position of comfort (typically a Fowler or semi-Fowler position). Fowler's is a bed-based position in which the head and trunk are raised, typically to 40-90°. If the patient is not breathing adequately, high-flow oxygen is appropriate to administer. Titrate as needed to maintain an SpO2 of 95-99%. Nitroglycerin is also used for patients with cardiac chest pain; however, vital signs would be required prior to administration of nitroglycerin since a contraindication is a systolic blood pressure of below 100 mmHg. An Automated External Defibrillator (AED) should be applied to a patient who is pulseless and unconscious.

Which of the following is not a common cause of altered mental status?

Pregnancy Hypertension is not a common cause of altered mental status. The mnemonic AEIOU-TIPS is often helpful to remember the possible causes of altered mental status: alcohol epilepsy, endocrine disorders, electrolyte imbalance insulin (i.e., hypoglycemia) opiates and other drugs uremia (kidney failure) trauma (particularly head trauma), temperature (hypothermia/hyperthermia) infection poisoning, psychogenic causes shock, stroke, seizure, syncope, space-occupying lesions, subarachnoid hemorrhage

Which of the following is defined as repeating a statement in order to ensure comprehension?

Reflection Reflection is restating a patient's statement to confirm your understanding. Empathy is being sensitive to a patient's feelings and thoughts. Facilitation is encouraging the patient to talk more or provide more information. Clarification is asking the patient to explain what they meant.

Which of the following is the most effective way to control disease transmission?

Proper handwashing Proper handwashing is the simplest yet most effective way to control disease transmission. Handwashing should be performed before and after contact, even if gloves are used. Skin breaks are potential access points for pathogens. Hands should be rinsed with warm water. If running water is not available, waterless hand-washing substitutes may be used. Gloves and eye protection are required for all patients when there is a risk of exposure to blood or body fluids. Change gloves between patients or if the gloves have been exposed to motor oil, gasoline, or any petroleum-based products. Gowns are commonly used for protection against extensive blood splatter. Gowns are often not practical in the field but are commonly used during the delivery of a baby or a major trauma. A standard surgical mask should be used if blood or body fluid splatter is possible.

Which of the following is the most successful ventricular fibrillation treatment modality that can be administered in a prehospital setting?

Rapid defibrillation An automated external defibrillator (AED) is indicated for any patient who is unresponsive and does not have a palpable pulse. Defibrillation will be advised if a patient has ventricular tachycardia or ventricular fibrillation; the defibrillation may remedy the arrhythmia and allow for a more stable rhythm. CPR is an important component of the treatment of cardiac arrest patients because it prolongs the period during which defibrillation can be effective. Assisted ventilation is required for any patient who is in respiratory distress/failure or experiencing apnea. It does not affect cardiac rhythm. Nitroglycerin is indicated for the ease of cardiac chest pain; it does not affect cardiac rhythm.

Which of the following movements is appropriate when arriving at a scene with suspected hazardous material?

Remain upwind and uphill When dealing with hazardous materials, scene safety is important. Remain upwind and uphill of the scene. Keep a safe distance, retreating if danger is noted. Contact dispatch and request additional resources. Do not enter the scene until it is deemed safe by trained hazardous materials experts. Leaving the scene may constitute abandonment, as patients may require care, and trained individuals have not been contacted to attempt to secure the scene.

A 22-year-old female has multiple facial injuries. The patient states that her husband assaulted her. She explains that this is not the first incident. While you are treating the patient, the husband returns and demands to be left alone with his wife. Which of the following is the appropriate next step?

Request police assistance Scene safety and patient safety are paramount. If a violent situation arises or seems possible, EMS should contact law enforcement for assistance. If possible, complete this request in a way that does not draw the attention of the violent individual. Confronting with arrest or ignoring the husband or asking the husband to leave the scene may further aggravate him, escalating the situation and increasing the likelihood of violence.

An 82-year-old male has confirmed cardiac arrest. After the AED application, a "no shock indicated" message is received. What is the appropriate next step?

Resume CPR, beginning with chest compressions CPR should be immediately resumed after a "no shock indicated" message is received or defibrillation is completed. A patient's pulse and breathing are reassessed after five cycles (two minutes) of CPR are completed. Most AEDs will instruct the assessment of a pulse at that time. CPR should not be delayed to attach a different AED.

A 50-year-old female is unresponsive, hypotensive, and breathing at a rate of four breaths per minute with an oxygen saturation of 88% on room air. Impending cardiac arrest is likely. A family member with appropriately documented power of attorney for health care over the patient authorizes only supportive care for the potentially dying patient. There is no documentation available, and several other family members on the scene are upset and begging for resuscitation. Which of the following would be most appropriate action in this scenario?

Resuscitate the patient. If there is ever a question about power of attorney for health care, medical direction should be consulted. A person with a power of attorney for health care can make decisions for patients when they are unable to make decisions for themselves. However, in this situation, there is no indication that a do-not-resuscitate order exists. If the power of attorney is questionable, emergency care should not be delayed. Withholding treatment measures would be negligent in this scenario.

A 10-year-old patient is in respiratory distress. During the assessment, the patient reports severe nausea. Which of the following is likely occurring?

Severe allergic reaction Most external allergic reactions remain localized. Anaphylaxis is a severe allergic or hypersensitivity reaction characterized by rapid onset and is potentially lethal. The diagnosis is based on clinical signs and symptoms, within the context of events in the preceding minutes or hours. Anaphylaxis can create bronchospasms, wheezing, chest tightness, coughing, dyspnea, hypotension, anxiety, and gastrointestinal complaints. Heat exhaustion commonly causes dizziness, weakness, syncope, muscle cramping, nausea, vomiting, headaches, clammy or ashen skin, a dry tongue, thirst, and tachycardia. An asthma attack commonly involves characteristic wheezing, cyanosis, and potentially respiratory distress/arrest. Aspirin overdose can result in nausea, vomiting, hyperventilation, tinnitus, anxiety, confusion, hyperthermia, and potential seizures.

Which of the following substances are examples of sedative-hypnotic drugs?

Sedative-hypnotic drugs depress the central nervous system and alter the level of consciousness. They do not treat pain directly but alter the perception of pain. Examples include diazepam (Valium), alprazolam (Xanax), lorazepam (Ativan), ketamine, and phenobarbital. Morphine treats pain directly and can cause effects similar to those of a sedative-hypnotic drug, but it is opiate-based and considered a narcotic analgesic.

Which of the following types of injuries would not require spinal motion restriction?

Shoulder injury Spinal motion restriction should be considered for patients with multisystem trauma injuries or suspected hip, pelvic, spinal, and lower extremity injuries. Follow local protocols, but spinal motion restriction may be accomplished with a C-collar. A shoulder injury does not necessarily require a long backboard. A shoulder injury is immobilized in a position of comfort and placed in a sling and swathe.

Which of the following are signs/symptoms of cholecystitis? Select the three answer options which are correct.

Signs and symptoms of cholecystitis include: upper-right-quadrant pain (especially after eating fatty foods) referred pain to the right shoulder nausea vomiting fever jaundice loose, light-colored bowel movements Dark, tarry stools are a sign of lower GI bleeding, not cholecystitis.

Which of the following demonstrates an appropriate way to communicate with an elderly patient?

Sir, I'm going to try to make you as comfortable as I can on your way to the hospital." When dealing with geriatric patients, speak respectfully. Avoid showing frustration or impatience through body language. Look directly at the patient at eye level. Speak slowly and distinctly. Ensure only one person is talking to the patient at a time and give the patient adequate time to respond. Listen to the answers from the patient. Do not assume all older patients are hard of hearing. Explain what assessments, treatments, or interventions are going to be done prior to completing them. Do not talk about the patient as if they were not there.

Which of the following is not a method of gauging a patient's responsiveness to painful stimuli?

Slapping the patient's cheek To assess a patient's level of consciousness if they are not responsive to verbal stimuli, it is appropriate to switch to painful stimuli. Appropriate painful stimuli include applying gentle but firm nail bed pressure, gently but firmly pinching the patient's ear lobe, pressing the bone above the eye, and gently but firmly pinching the neck muscles. Striking a patient is inappropriate.

While you are evaluating a patient with an altered mental status from suspected alcohol ingestion, he becomes agitated, verbally abusive, and physically aggressive. He states he just wants to be left alone. Which of the following is the appropriate next step?

Step back and prepare to retreat, if necessary, while awaiting assistance from law enforcement Personal safety is paramount on every call. This patient may hurt EMS personnel if given the opportunity. If there is a concern that a patient may physically assault any member of the EMS team, move away from the situation and contact law enforcement. An alert, competent adult may refuse treatment. However, the risks, benefits, and alternatives to the treatment and its refusal must be explained to the patient prior to accepting a refusal. Any refusal of treatment/transport should be documented and signed by the patient, with a witness. However, in most states, patients who are impaired in any way (due to mental illness, a medical condition, or intoxication) cannot legally refuse treatment/transport. Leaving in this scenario may constitute abandonment.

A patient has a partially obstructed airway. Which of the following findings is expected?

Stridor Wheezing, stridor, and coughing are commonly heard in patients with a mild/partial airway obstruction. A patient with partial airway obstruction should be monitored for adequate oxygenation and progression of the obstruction. Patients with a partial airway obstruction and poor air exchange may present with an ineffective cough, stridor, increased difficulty breathing, and cyanosis. If the obstruction is visible, use a gloved index finger to sweep the obstruction forward and remove it from the mouth. Suction may be used to assist with this procedure as well as to maintain a clear airway. If the obstruction is not visible and air exchanged is adequate, administer oxygen and rapidly transport. A patient with poor air exchange should be treated as if they have a severe airway obstruction. Those with a severe airway obstruction will be unable to breathe, talk, or cough. Cyanosis and extreme difficulty with breathing are common. Unconsciousness is possible. Opening the airway and performing abdominal thrusts are appropriate for a severe airway obstruction. Do not attempt to remove a foreign body if it cannot be visualized. Crackles are high-pitched (fine) or low-pitched (coarse) crackling or bubbling sounds heard on inspiration. They result from air trying to pass through fluid in the alveoli. These sounds are often indicative of pulmonary edema or congestive heart failure.

An error was discovered after the submission of a patient care report to a receiving hospital. Which of the following is the most appropriate step?

Submit an addendum with the new information Errors are likely to occur over an EMS member's career. If information was omitted from a patient care report, an addendum should be created with the addition of the new information, the date, and the EMS member's initials. It is not necessary to retrieve the original report. It is inappropriate to ignore any mistake or error in documentation. Filing a report with a supervisor does not correct the omitted information.

A 34-year-old male is agitated and confused. Which of the following signs would indicate cocaine use?

Tachycardia Sympathomimetics (e.g., cocaine, amphetamine/methamphetamine, epinephrine, albuterol) are stimulants that mimic the effects of the sympathetic nervous system. Common signs and symptoms of use/overdose include hypertension, tachycardia, dilated pupils, agitation, seizures, and hyperthermia. It should be noted that anticholinergics (e.g., atropine, diphenhydramine) have similar signs and symptoms. Pinpoint pupils are generally only seen with opiate (e.g., morphine, heroin, methadone, oxycodone) use or overdose. Hypotension and sedation may be seen with opiate or sedative-hypnotic (e.g., diazepam) use.

The AHA's "Hs and Ts" mnemonic summarizes the possible reversible causes of cardiac arrest in an adult patient. Which of the following are included in this mnemonic?

The AHA's Hs and Ts include: hypovolemia hypoxia hypoglycemia hydrogen ion (acidosis) hypokalemia/hyperkalemia hypothermia tension pneumothorax tamponade (cardiac) toxins thrombosis

The GEMS diamond mnemonic was created to help EMS providers recall key concepts about the assessment of geriatric patients. Which of the following are categories of the GEMS diamond?

The GEMS diamond assessment system is a simple way to recall elements that are key to working with older patient groups. Most importantly, remember older patients have thoughts, ideals, and motivations just like their younger cohorts. G: geriatric patient E: environmental assessment M: medical assessment S: social assessment

Which of the following are events that would cause the EMT to stop CPR?

The STOP mnemonic helps EMTs know when to stop CPR. S: The patient starts breathing and has a pulse. T: The patient's care is transferred to another provider of equal or higher level. O: You are out of strength and can no longer physically continue CPR. P: A physician orders the termination of CPR, either in person or online. A "no shock advised" message by the AED means the patient is not in a shockable rhythm, CPR should continue.

Which of the following options best explains the process of inhalation?

The diaphragm and intercostal muscles contract, increasing the size of the thoracic cavity and creating a negative pressure, which pulls air into the lungs. Inhalation is an active process. When the diaphragm and intercostal muscles contract, the size of the thoracic cavity increases, creating a negative-pressure space (i.e., lower than atmospheric pressure). Exhalation is a passive process. When the diaphragm and intercostal muscles relax, the size of the thoracic cavity decreases, which pushes air out of the lungs.

Which of the following conditions are endocrine system disorders?

The endocrine system is a complex messaging system that uses hormones and endocrine glands. Diabetes is an endocrine disorder that affects the pancreas and the hormone insulin. Addison's disease and Cushing's disease are endocrine disorders caused by pituitary gland and adrenal gland abnormalities. Graves disease is an endocrine disorder caused by thyroid gland abnormalities. GERD (Gastroesophageal Reflux Disease) is a disorder of the gastrointestinal system.

During a secondary assessment on an adult trauma patient, what is the first area to be assessed?

The head For an adult trauma patient, a secondary assessment (head-to-toe) begins at the head and moves inferiorly. The face, including the eyes and pupil response, is the first to be assessed. The remainder of the head and neck follows. After the neck is assessed, the torso, including auscultation of breath sounds, is done; if possible, auscultate the posterior breath sounds at this time. The abdomen, pelvis, and lower extremities are then assessed. Finally, the patient is log-rolled to one side, and the back is inspected; posterior breath sounds may be auscultated at this time if not previously done.

An adult male patient had a syncopal episode and was evaluated by EMS personnel at his home. He was alert with pale, dry skin. He refused treatment and transport. Later that same day, he experienced a cardiac arrest and died. Which of the following statements is correct regarding this scenario?

The patient has the right to self-determination and to make decisions about his care. An alert, competent adult patient may refuse treatment. However, the risks, benefits, and alternatives to the treatment and its refusal must be explained to the patient prior to accepting a refusal. Any refusal of treatment/transport should be documented and signed by the patient with a witness. EMS personnel would not be liable in this situation as long as the patient was fully aware at the time of the consequences of his actions. A duty to act is the responsibility to provide patient care. This may occur if a person works as an emergency medical responder or their service or department's policy states that assistance must be provided in an emergency. A duty to act begins once a response has been assigned/initiated, and it ends when a transition of care occurs or a patient refuses care.

Which of the following requirements must be met before an analysis of a patient's heart rhythm with an automated external defibrillator (AED)?

The patient is pulseless and unresponsive. An AED is indicated for a patient who is pulseless and unresponsive. An AED should not be used on a patient who is responsive or has a palpable pulse. A history of heart disease is not an indication for or a contraindication to AED use.

What are the 3 parts of the pediatric assessment triangle?

The pediatric assessment triangle was developed to help EMS providers gain a "from the doorway" general impression of pediatric patients. It is composed of appearance, work of breathing, and circulation to the skin.

Research has identified the "trauma lethal triad" as a major contributor to death in patients with severe traumatic bleeding. What are the compon

The trauma lethal triad consists of hypothermia, coagulopathy, and acidosis. These are considered major contributors to death in patients with severe traumatic bleeding. Knowing the components of the triad helps EMTs remember important treatment aspects of the patient with severe blood loss: keep the patient warm, provide oxygen/assisted ventilation as needed, and arrange rapid transport to a trauma center. Hypertension, alkalosis, and hyperthermia are not components of the trauma lethal triad and are not a common presentation for a patient with traumatic blood loss.

Which of the following are considered endocrine emergencies?

Thyroid storm, diabetic ketoacidosis (DKA), and hyperosmolar hyperglycemic nonketotic syndrome (HHNS) are the result of thyroid gland and pancreas abnormalities, which are parts of the endocrine system. Sickle cell disease affects red blood cells, which are a part of the circulatory system.

What is the purpose of an incident command system?

To ensure responder and public safety, achieve incident management goals, and promote the efficient use of resources An incident command system (ICS) ensures responder and public safety, achieves incident management goals, and promotes the efficient use of resources. An ICS is used in mass-casualty incidents (three or more patients). Depending on the size of the incident, multiple organizations may be involved. Clear communication is key to a functional ICS. Most incidents are local, but in a worst-case scenario like a mass-casualty event, all responding agencies must be able to interface and work together. The NIMS and, in particular, the ICS component, allow this response to happen, but only if the foundation has been laid at the local level. If local jurisdictions adopt a variation of ICS that cannot grow or is not applicable to other disciplines, the critical interface between responding agencies and jurisdictions cannot occur when the response expands. Among other things, the National Incident Management System (NIMS) is responsible for standardizing incident management for all hazards across all levels of government. An ICS is not involved in the evaluation of an EMS system or the training of first responders; instead, an ICS provides guidance, direction, and order during a larger incident.

A 56-year-old male is hypothermic and unresponsive. Which of the following is the main goal of this patient's prehospital emergency care?

To prevent further heat loss For patients with moderate or severe hypothermia, the main goal is to prevent further heat loss. This is done by removing the patient from the cold environment, removing wet/restrictive clothing, covering the patient with dry blankets, and placing the patient in a warm environment. Ensure that the patient is handled with care to prevent further injury and transport rapidly. Warm, humidified oxygen may also be used. The patient should not attempt to walk, as this may cause further damage to the feet. Attempting to rewarm as fast as possible may cause tissue damage or worsen symptoms. Active rewarming should be completed in the hospital with constant core body temperature monitoring.

What is the purpose of the head-tilt/chin-lift technique?

To relieve an obstruction blocking an otherwise-patent airway when spinal trauma is not suspected The head-tilt/chin-lift maneuver is the primary method used to open the airway of non-trauma patients. Possible obstructive objects include the tongue, foreign objects (e.g., food, small toys, teeth/dentures), blood clots, damaged tissues, and aspirated vomitus. The head-tilt/chin-lift maneuver is completed by placing the patient in a supine position and the rescuer beside the patient's head. The heel of one hand is placed on the patient's forehead, and firm pressure is applied to tilt the patient's head back. The fingertips of the alternate hand are placed under the lower jaw, near the bony part of the chin; the chin is then lifted upward. The forehead is held to maintain the backward tilt. If spinal trauma is suspected, the jaw-thrust maneuver is performed. The rescuer kneels above the patient's head with fingers behind the angles of the lower jaw. The jaw is then moved upward with the index and middle fingers, and the thumbs help position the lower jaw. A patient is log-rolled to the side to help assist in the removal of foreign bodies. Only visible foreign bodies are to be removed. Inline stabilization of the cervical spine is used to ensure no further damage occurs until a C-collar can be applied.

What is the main purpose of a detailed physical examination?

To reveal any hidden or less obvious injuries that may not have been found on the initial assessment The purpose of the secondary assessment is to perform a systemic or focused physical examination of the patient. Depending on the mechanism of injury or nature of the illness, the secondary assessment may be a complete examination or focus on the chief complaint. In adult patients, a complete examination begins at the head and proceeds inferiorly. In children, a complete examination begins at the feet and moves superiorly. Part of the purpose of a scene size-up is to ensure the safety of EMS personnel, patients, and bystanders. The purpose of the primary assessment is to identify and begin treatment of immediate or imminent life-threatening conditions. The purpose of a reassessment is to identify and treat changes to the patient's condition.

When the right ventricle contracts, how does the blood that is contained in the ventricle flow?

To the lungs via the pulmonary arteries Deoxygenated blood is pumped from the right ventricle to the lungs via the pulmonary arteries. Oxygenated blood returns from the lungs to the left atrium via the pulmonary veins. Oxygenated blood is then pumped from the left ventricle to the body via the aorta. Coronary arteries branch from the aorta and feed oxygenated blood to the cardiac muscles. Deoxygenated blood flows from the body to the right atrium via the superior and inferior vena cavae.

How far should a rigid tip suction catheter be inserted into the mouth?

To the pre-measured depth An EMT should make sure that the catheter only goes as far as the pre-measured depth. (Measure from the corner of the mouth to the angle of the jaw.) The catheter should never go farther than the base of the tongue; suction only the mouth and oropharynx. An EMT should make sure that the catheter never goes farther than the base of the tongue; this is well past the last (third) molar. If a suction catheter tip reaches the epiglottis or vocal cords, it has been inserted too far.

Which of the following is the opposite of urinary incontinence?

Urinary retention Incontinence is the inability to postpone voiding or bowel movements. If a patient is unable to void, they are suffering from urinary retention. A common cause of urinary retention in males is an enlarged prostate. If urinary retention progresses, renal failure may occur. Cystitis is a urinary tract infection of the bladder. These infections may cause inflammation and potential renal damage as well. Kyphosis is the forward curling of the back caused by an increased curvature of the spine. Renal calculi are kidney stones. If a stone blocks a ureter, pressure will build behind the stone, and the kidney will swell.

A 22-year old single mother fractured her ankle while playing with her children. Her leg is clearly deformed, and she reports a pain level of 9/10. Her children, ages 2 and 5, are frightened but cooperative. The patient agrees to transport if her children can accompany her, as no family members or neighbors are available to care for them. Which of the following is the best option?

Transport the children using appropriate restraint systems in an accompanying police vehicle While family-centered care is desirable, it must be done without placing the patient, family, child, or provider at increased risk. When possible, arrange for alternative care and/or transportation so that attention can be focused on the patient. Appropriate child restraints should be used when possible, although ambulances are often ill-equipped to secure pediatric passengers. If the child is not the patient, consider utilizing accompanying law enforcement or another emergency vehicle. If separating the child and parent is problematic, follow proper procedures and protocols and secure both patients with appropriate restraint systems and positions. There is no way to secure a car seat safely to an ambulance bench. Stretchers are designed for securing one patient only. A taxicab is unlikely to have an appropriate restraint system for infants or toddlers, and transporting unaccompanied children is inappropriate. It is never acceptable to secure a child in a parent's arms for ambulance transport.

A semiconscious adult patient requires suctioning due to copious amounts of mucus without likely foreign bodies in the oropharynx. What is the most appropriate method of suctioning?

Turn the head to the side (if no spine injury is suspected) Suctioning removes oxygen and can result in hypoxia. It is important to insert the catheter no further than can be visualized and to suction during withdrawal for no more than 10 seconds for adult or pediatric patients. The suctioning unit should have at least 300 mmHg of vacuum pressure. Prolonged suctioning can actually obstruct an airway. Aggressive suctioning may cause vagal stimulation, especially in children, which may cause further hypoxia or bradycardia. It is recommended to limit suction attempts for patients of all ages to 10 seconds. Some textbooks may reflect 15 seconds, but the current recommended practice is 10 seconds.

How many lobes are there in a normal left lung?

Two The human body has two lungs. The right lung is divided into three lobes (upper, middle, and lower), while the left lung has two lobes (upper and lower).

Which of the following is not an example of a mass-casualty incident?

Two patients in cardiac arrest A mass-casualty incident (MCI) is a situation that involves three or more patients, places a great demand on the EMS system, and/or has the potential to produce multiple casualties. Disasters are common examples of MCIs; alternatively, a residential building fire confined to one apartment may only produce one patient but has the potential to spread to dozens of patients. Additionally, loss of power to a hospital or nursing home with ventilator-dependent patients can be considered an MCI, even though no one is injured. Each location and system will have different protocols for when to declare an MCI. If additional resources are required, dispatch may be contacted. The two patients in cardiac arrest can be managed between two EMS units or an EMS unit with three responders. Per the 2019 AHA Guidelines Update on Systems of Care, emergency dispatch centers should offer CPR instructions and empower dispatchers to provide such instructions for adult patients in cardiac arrest. Dispatchers should instruct callers to initiate CPR for adults with suspected out-of-hospital cardiac arrest. This guideline is associated with approved outcomes in out-of-hospital cardiac arrest and has a five-fold increase in bystander CPR. An active fire has the potential to injure anyone in the apartment building. This multi-car crash has more than three patients. While no one in the bleacher collapse shows signs of obvious deformities, internal injuries are possible; all patients should be assessed.

Which of the following is not an appropriate technique for stress reduction?

Using sedatives to sleep Stress is any event, thought, or action that is perceived as a threat. It is imperative to manage stress appropriately in order to promote personal safety, scene safety, and patient care. Some strategies to help manage stress include: minimizing/eliminating stressors as much as possible avoiding negative/hostile personalities changing work hours cutting back on overtime changing work environments changing attitudes about stressors talking with trusted people seeking professional help, if needed not obsessing over situations that are out of one's control adopting a more relaxed outlook expanding one's social support system beyond coworkers stretching/yoga slow, deep breathing regular physical exercise progressive muscle relaxation meditation limiting caffeine, alcohol, and tobacco use Muscle relaxers, tranquilizers, and sedatives have legitimate uses, but they do not combat stress, may cause medical problems, and have addictive properties.

Which major blood vessel carries deoxygenated (oxygen-poor) blood to the heart?

Vena cava Deoxygenated blood flows from the body to the heart via the superior and inferior vena cava. Deoxygenated blood is pumped from the heart to the lungs via the pulmonary artery. Oxygenated blood returns from the lungs to the heart via the pulmonary vein. Oxygenated blood is then pumped from the heart to the body via the aorta.

Which of the following signs is not an indicator that a patient may be experiencing inadequate breathing?

Warm, dry skin with normal capillary refill Inadequate breathing leads to inadequate oxygen perfusion, which may cause bradypnea or tachypnea in the presence of shortness of breath (dyspnea); irregular breathing rhythm; diminished, absent or noisy auscultated breath sounds; unequal or inadequate (shallow) chest expansion/depth; use of accessory muscles; cyanosis; cool or moist skin; or skin pulling in around the ribs (retractions). Adventitious breath sounds are abnormal sounds such as crackles or rales, wheezes or rhonchi, pleural rubs, or stridor.

A 10-year-old male is found unconscious from an unknown mechanism of injury. Which of the following is the most appropriate method to open his airway?

jaw-thrust maneuver The appropriate methods for opening an airway in a prehospital setting are the head-tilt/chin-lift maneuver and the jaw-thrust maneuver. If trauma is suspected, the jaw-thrust maneuver is the most appropriate method to open an airway. Do not modify the head-tilt/chin-lift method unless necessary. The sniffing position is when a patient spontaneously flexes their neck forward and extends their head up and back in order to open a partially obstructed upper airway. It is not appropriate for a trauma patient due to the risk of cervical spine injury.

An 8-month-old infant becomes unresponsive at a preschool facility. The infant has a palpable pulse of 46 beats per minute without notable respirations. Which of the following is the most appropriate next step?

nitiate CPR, beginning with chest compressions CPR should be initiated on an infant without a palpable pulse or a pulse below 60 beats per minute; chest compressions are started before ventilation in CPR. The ratio of chest compressions to ventilations with two-rescuer CPR for infants is 15:2. A patient who is not adequately breathing and has a palpable pulse should be given assisted ventilation; the rate of assisted ventilation for a child is a breath every three to five seconds (12-20 breaths per minute). Children in this scenario should be placed in a position that allows for the management of the airway. High-flow oxygen would not be appropriate for a patient who is not breathing adequately.

An 18-year-old patient was involved in a motor vehicle crash. Upon examination, an obvious deformity is noted in the left mid thigh, indicating a femur fracture. Which of the following splints would be most appropriate?

traction splint A traction splint provides in-line traction, the act of pulling on a body structure in the direction of its normal alignment. Traction splints are used only to secure mid-shaft fractures of the femur. This provides stabilization of the fracture and alignment of the limb to reduce pain and potential neurovascular compromise. Contraindications for traction splint use include injuries to the pelvis, hip fractures, multiple fractures of the leg (knee injury, lower leg, foot, or ankle). Traction splints are not to be used for the upper extremities. An air splint is used to stabilize injuries below the elbow or knee. A rigid splint may be used, but barring contraindications, a traction splint may provide better pain relief and protection from further injury. A pelvic binder offers temporary stabilization of the pelvis to reduce hemorrhage from bone ends, venous disruption, and pain.

Which of the following is a contraindication for the use of nitroglycerin?

ystolic blood pressure below 90 mm Hg Nitroglycerin is a vasodilator that is indicated for patients with cardiac chest pain. Medical direction must be contacted, and the "six rights" must be checked before assisting in the administration of this medication. Nitroglycerin is contraindicated in patients who: have a systolic blood pressure below 90 mm Hg have a heart rate below 60 have not been prescribed this medication are pediatric (under 13 years of age) have taken medication for pulmonary artery hypertension within the previous 48 hours (Adcirca, Revatio) have taken medication for erectile dysfunction within the previous 48 hours (Viagra, Levitra, Cialis) have already taken their maximum dose (typically three doses). Most patients are allowed to repeat doses (typically up to three) if the pain remains after five minutes; because of the previously mentioned contraindication, blood pressure should be assessed before administration of initial or repeat nitroglycerin.


Kaugnay na mga set ng pag-aaral

Adolescence: Chapter 3 Brain and Cognitive Development

View Set

Chapter 4 Review - The Pesticide Label

View Set

US History- WWII Review Questions

View Set

Mental Health Chapter 21 Evolve Questions (Impulse Control Disorders)

View Set

PTH523: Module 13: Knee and Popliteal Fossa

View Set

History Unit 1 Global Tapestry copy

View Set

Principles of Marketing - Part 3 of 4

View Set